Trichomonas vaginitis (TV) MCQs

अब Quizwiz के साथ अपने होमवर्क और परीक्षाओं को एस करें!

A 22-year-old woman presents with intensely irritating yellowish-green frothy vaginal discharge with severe dyspareunia. The organism is seen best under the microscope in a drop of saline. Which is the most likely causative organism? A. Treponema pallidum B. Trichomonos vaginalis C. Klebsiella granulomatis D. Neisseria gonorrhoeae E. Phthirus pubis

B. Trichomonos vaginalis Explanation: Trichomoniasis is a sexually transmitted infection caused by the flagellated protozoan Trichomonas vaginalis, which invades superficial epithelial cells of the vagina, urethra, glans penis, prostate and seminal vesicles. Affected women present with an offensive greeny-grey discharge, vulval soreness, dyspareunia, dysuria, vaginitis and vulvitis. On examination, the cervix may have a punctate erythematous appearance ('strawberry cervix'). Men are mostly asymptomatic. Treatment is with metronidazole. Syphilis is caused by the bacterium Treponema pallidum and is spread by sexual contact (it can also be acquired congenitally). There are many stages of syphilis infection: Primary syphilis, Secondary syphilis, Latent syphilis and Tertiary syphilis. Initial diagnosis of syphilis is by dark-ground microscopy (which shows the bluish coiled treponema organisms against the dark-brown background) and the Venereal Disease Research Laboratory (VDRL) test, which detects the presence of anticardiolipin, an antibody produced by people with syphilis. False positives to the VDRL test can occur with infectious mononucleosis, antiphospholipid syndrome and leprosy. Treatment of syphilis is with benzylpenicillin (or doxycycline if there is a penicillin allergy). Granuloma inguinale (donovanosis) is caused by the bacterium Klebsiella granulomatis. A flat-topped papule develops on the genitalia (days to months postinfection), and then degenerates into a painless ulcer. The ulcer spreads along skinfolds, and heals with scarring. Gonorrhoea is caused by the Gram-negative diplococcus Neisseria gonorrhoeae. The majority of infected women are asymptomatic, but some complain of vaginal discharge and urethritis. Complications include Bartholin's abscess and gonococcal salpingitis with irreversible tube damage. Infected men present with dysuria, frequency and/or a mucopurulent discharge after 3-5 days, coupled with urethritis and meatal oedema. Disseminated gonococcal infection occurs in <1% cases and causes pyrexia, a vasculitic rash and polyarthritis. Gonorrhoeal infection is confirmed by culture (Gram-negative diplococci), and treatment is with appropriate antibiotics (e.g. ciprofloxacin). Phthiriasis ('crabs') is caused by the crab-louse Phthirus pubis, which lives mainly in the thick hairs of the pubic and perianal areas. Most cases are due to sexual transmission, but any close contact with an infected person can transmit the lice. Clinically, there is itching in the affected areas. The lice feed on blood, and can leave spots on the skin (pediculosis pubis). Phthirus, from Greek phtheir = louse.

A sexually transmitted infection commonly presenting with vulval discharge and itching, dysuria and offensive odour. Overall, 10-50% of women are asymptomatic. Of women that are infected, the urethra is colonised in 90%, and 2% of women will have a 'strawberry cervix'. Which of the following is the most likely causal organism? A. Chlamydia trachomatis B. Human papillomavirus C. Molluscum contagiosum virus D. Mycoplasma genitalium E. Trichomonas vaginalis

E. Trichomonas vaginalis Explanation:

A 27-year-old woman presents to her GP with vaginal discharge, dysuria and vulval irritation. Examination of the vulva shows purulent frothy malodorous discharge and punctate haemorrhage on the cervix. What is the most likely diagnosis? A. Candidiasis B. Schistosomasis C. Bacterial vaginosis D. Syphilis E. Trichomoniasis

E. Trichomoniasis Explanation: Strawberry cervix with its characteristic vascular pattern is only present in 2% of the cases although this may be more visible at colposcopy.

Trichomonas vaginalis infection in pregnancy is associated with preterm birth and low birth weight. What percentage of infected women are asymptomatic? A. 10-50% B. 50-60% C. 60-70% D. 70-80% E. 80-90%

A. 10-50% Explanation: Trichomonal vaginalis is associated with preterm birth and low birth weight. 10-50% of women infected are asymptomatic. Trichomonasis is the most common non viral STI worldwide. Nitroimidazoles are the most effective treatment for Trichomonal vaginalis. There are insufficient safety data for tinidazole in pregnancy and its use cannot be recommended. A test of cure is recommended if symptoms fail to resolve or if symptoms recur.

You see a patient in clinic who complains of vaginal soreness and discharge. Examination reveals vulval irritation and a "strawberry" cervix. A wet smear is sent for microscopy which confirms trichomoniasis. What percentage of trichomoniasis cases would you expect to see a strawberry cervix? A. 2% B. 10-20% C. 50% D. 70% E. >90%

A. 2% Explanation:

Your patient has vaginitis caused by the microorganism Trichomonas vaginalis. Four 500-mg tablets of metronidazole have been prescribed. Which of the following is the correct instruction for the patient when she arrives to pick up the medication? A. Take all 4 tablets at once B. Take 1 tablet a day for 4 days C. Consume alcohol only in moderation while taking this medicine D. This medication is for symptomatic relief only; there is no cure for this infection E. Sexual contact can resume in 48 hours

A. Take all 4 tablets at once Explanation:

Neonatal Trichomonas may develop perinatally as a result of passing through the vagina of an untreated infected mother. What percentage of exposed babies would develop vulvovaginitis? A. 5% B. 10% C. 15% D. 20% E. 25%

A. 5% Explanation: Trichomonasis in pregnancy is associated with premature rupture of membranes, preterm delivery and low birth weight. Although perinatal transmission of trichomoniasis is uncommon, treatment also might prevent respiratory or genital infection of the newborn. Trichomonas vaginalis may be acquired perinatally, occurring in 5% of babies born to infected mothers. Infection beyond the first year of life suggests sexual contact and this should be investigated. Nitroimidazoles are the most effective treatment for Trichomonal vaginalis. • The BNF recommends metronidazole by mouth in children with urogenital trichomonasis as follows: ▶ Child 1-2 years: 50 mg 3 times a day for 7 days. ▶ Child 3-6 years: 100 mg twice daily for 7 days. ▶ Child 7-9 years: 100 mg 3 times a day for 7 days. ▶ Child 10-17 years: 200 mg 3 times a day for 7 days, alternatively 400-500 mg twice daily for 5-7 days, alternatively 2 g for 1 dose. • The BNF recommends tinidazole by mouth in children with urogenital trichomonasis as follows: ▶ Child 1 month-11 years: 50-75 mg/kg (max. per dose 2 g) for 1 single dose, dose may be repeated once if necessary. ▶ Child 12-17 years: 2 g for 1 single dose, dose may be repeated once if necessary.

You see a patient in clinic who complains of vaginal soreness and discharge. Examination is unremarkable. High vaginal swab is taken from the posterior fornix, and microscopic examination reveals a trichomonas vaginalis. What proportion of women will experience a normal examination? A. 5-15% B. 15-30% C. 30-45% D. 45-60% E. 60-75%

A. 5-15% Explanation: Trichomoniasis, a sexually transmitted infection of the urogenital tract, is a common cause of vaginitis in women, while men with this infection can display symptoms of urethritis. 'Frothy', greenish vaginal discharge with a 'musty' malodorous smell is characteristic. 10-50% of women have no symptoms and 5-15% of women have a normal examination. Only 2% of women with the infection will have a "strawberry" cervix (colpitis macularis, an erythematous cervix with pinpoint areas of exudation) or vagina on examination. This is due to capillary dilation as a result of the inflammatory response.

A woman in the outpatient clinic tells you that she has been diagnosed with Trichomonas vaginalis. What kind of organism is Trichomonas vaginalis? A. Flagellated protozoan B. Intracellular diplococcus C. Phthiriasis D. Pox virus E. Yeast

A. Flagellated protozoan Explanation: Trichomonas vaginalis is a flagellated protozoan which is best diagnosed when a swab is used to sample the fluid in the posterior fornix of the vagina and examined on a slide using a microscope as a "wet prep". Women typically present with a malodorous "frothy" discharge. Trichomonas vaginalis is a flagellated protozoan, thus whilst intracellular diplococcus, phthiriasis and pox virus describe other types of sexually transmitted infections, only flagellated protozoan correctly categorises Trichomonas vaginalis. Phthiriasis or Phthirus pubis describes pubic lice, which can be diagnosed with the naked eye in the pubic hair. Appropriate treatment includes malathion lotion and permethrin cream (unlicensed). All extra-genital body hair should also be treated and all bedding should be washed at a high temperature. Sexual partners should also be notified and treated. Candida albicans is yeast which may be identified by culture or by the presence of hyphae, pseudohyphae and spores on microscopy. Current evidence does not suggest that candida is sexually transmitted. Neisseria gonorrhoea is a Gram negative intracellular diplococcus. Molluscum contagiosum is a pox virus which is transmitted by skin-to-skin contact and often presents with umbilicated smooth papules in the genital region.

A 17-year old female presents with complaints of itching in her genital region. A wet mount indicates a Trichomonas vaginalis infection. Which of the following can be used to treat her condition? A. Metronidazole B. Mebendazole C. Mefloquine D. Niclosamide E. Niridazole

A. Metronidazole Explanation: Metronidazole is the drug of choice. Mebendazole (B) is a broad spectrum antinematode agent. Mefloquine (C) is used as a prophylactic drug to prevent malaria and also used to treat chloroquine-resistant clinical malaria. Niclosamide (D) is a broad-spectrum agent effective in the treatment of adult tapeworm infections. Niridazole (E) is a drug used to treat schistosomiasis if praziquantel is not available.

A 22-year-old woman presents to the physician with vaginal itching and burning. On examination, she has a foul-smelling greenish discharge. A swab sample is taken and a wet mount slide is prepared; results are shown in the image. Which of the following medications should be prescribed for this patient? A. Metronidazole B. Nifurtimox C. Quinine D. Sodium stibogluconate E. Sulfadiazine and pyrimethamine

A. Metronidazole Explanation: Trichomonas vaginalis is the cause of vaginitis. Symptoms of vaginitis include a foul smelling greenish discharge, itching, and burning. T. vaginalis is transmitted sexually. On microscopic wet mount, one finds trophozoites. Metronidazole is used to treat T. vaginalis infection. Answer B is incorrect. Nifurtimox is used to treat Trypanosoma cruzi. T. cruzi infection causes Chagas' disease, a condition in which the heart is enlarged and flaccid. T. cruzi is transmitted via the reduviid bug. Microscopic examination reveals flagellated trypomastigotes in the blood and non motile amastigotes in tissue culture. Answer C is incorrect. Quinine is used to treat babesiosis. Babesia species present with malaria- like syndrome. Babesiosis is transmitted by the Ixodes tick. On microscopic examination, one observes no red blood cell pigment and the Maltese cross-appearing parasite. Answer D is incorrect. Sodium stibogluconate is used to treat Leishmania donovani infection. L. donovani presents with hepatomegaly and splenomegaly, malaise, anemia, and weight loss. L. donovani is transmitted via the sand fly. Microscopically, macrophages containing amastigotes are observed. Answer E is incorrect. Sulfadiazine and pyrimethamine are used to treat toxoplasmosis. Toxoplasma gondii infection presents with brain abscesses in HIVpositive patients and with birth defects. T. gondii is transmitted via cysts in raw meat or cat feces. The definitive stage (sexual stage) occurs in cats. Microscopically, acidfast staining cysts are found.

A 25-year-old woman comes to the outpatient office complaining of a pruritic, painful vaginal discharge. She is sexually active with two male sexual partners but finds intercourse very uncomfortable because of her vaginal symptoms. For the past 8 months, she has been using the estrogen-progestin contraceptive patch. She exercises regularly by walking 2 to 3 miles a day. She is following a low-carbohydrate diet and takes a multivitamin preparation. Findings of her general examination are unremarkable. Speculum examination of the vagina shows a foul-smelling greenish, frothy discharge. Vaginal pH, using Nitrazine paper, is 6.5. A wet mount of vaginal secretions in a saline suspension reveals a highly motile organism. Which of the following pharmacologic agents would be the most appropriate treatment? A. Metronidazole B. Clotrimazole C. Miconazole D. Acyclovir E. Azithromycin

A. Metronidazole Explanation: The cause of the vaginitis is Trichomonas vaginalis, as evidenced by the pruritic discharge and the highly motile protozoan. The elevated vaginal pH and frothy greenish discharge are frequent clinical findings with trichomoniasis. The agent of choice is metronidazole (choice A). Due to an Antabuse effect of this medication, she should be advised not to drink alcohol while taking it. She also needs to be informed this is a sexually transmitted disease and her sexual partners also need to be treated. Ninety percent of females are symptomatic, compared with only 50% of males. Clotrimazole (choice B) and miconazole (choice C) are topical intravaginal antifungal agents used to treat vaginal yeast infections. Acyclovir (choice D) is an antiviral agent used in genital herpes management. Azithromycin (choice E) is an antibacterial macrolide antibiotic administered orally for the treatment of Chlamydia.

A 25-year-old woman presents to the office with a 1-week history of vaginal discharge. She describes the discharge as being green-yellow in color with a bad odor. She complains of increased vaginal soreness and discharge after she has intercourse. She is currently sexually active and is using an intrauterine device (IUD) as her contraceptive method. She has been with one male partner for the past 3 months and he has no symptoms. She had a chlamydial infection 2 years ago that was treated with oral antibiotics. Her last menstrual period was 2 weeks ago and was normal. On examination, she has normal external genitalia. She has a small amount of frothy, homogenous, green-gray discharge at the introitus. The cervix has a "strawberry"- red appearance with a slight amount of discharge noted in the os. The IUD string is in place. Bimanual examination shows no cervical motion tenderness, and normal uterus and adnexa. Chlamydia and gonorrhea specimens are obtained from the os. A wet-mount microscopic examination of the discharge shows motile, flagellated organisms, and many white blood cells. What is the best initial treatment for this patient? A. Metronidazole 2 g orally once B. Doxycycline 100 mg orally twice daily for 7 days C. Azithromycin 1 g orally once D. Clindamycin 300 mg orally twice daily for 7 days E. Metronidazole gel 0.75% one applicator intravaginally for 5 days

A. Metronidazole 2 g orally once Explanation: A single-dose regimen with metronidazole or tinidazole is the mainstay treatment of trichomoniasis. An alternative multidose regimen is metronidazole 500 mg orally twice daily for 7 days. Compliance may be improved with the single-dose therapy. Doxycycline (B) can be used to treat chlamydial infections. Azithromycin (C) can be used to treat chlamydial infections and chancroid. Clindamycin (D) can be used as an alternative regimen for the treatment of bacterial vaginosis. Metronidazole gel (E) is considerable less efficacious for the treatment of trichomoniasis. The use of topical antimicrobial preparation is unlikely to achieve therapeutic levels in the perivaginal glands and is not recommended.

A 39-year-old woman presents to clinic with a 5-day history of abnormally increased and slightly malodorous vaginal discharge. She has no dysuria or polyuria. She has been sexually active with two men in the past month and reports that she does not routinely use condoms. Point of care testing for both HIV and pregnancy are negative in clinic. Evaluation for bacterial vaginosis (using Amsel criteria) yielded only an abnormally elevated pH level. Vaginal swabs are sent for nucleic acid amplification testing for sexually transmitted infections and are negative for Chlamydia trachomatis and Neisseria gonorrhoeae, but positive for Trichomonas vaginalis. Which one of the following represents the best option to treat this woman's infection? A. Metronidazole 2 grams orally in a single dose B. Metronidazole 500 mg twice a day for 10 days C. Nitazoxanide 500 mg twice a day for 3 days D. Metronidazole gel intravaginally daily for 7 days E. Clindamycine gel intravaginally daily for 7 days

A. Metronidazole 2 grams orally in a single dose Explanation: This woman with symptomatic T. vaginalis infection should receive treatment. In the United States, only oral metronidazole and tinidazole are recommended as therapy for T. vaginalis infection. Treatment with these agents has demonstrated cure rates consistently above 90%. A single 2-gram dose of either metronidazole or tinidazole should be prescribed. Twice daily dosing of metronidazole for 7 days is an alternative regimen, but single-dose treatment is preferred in uncomplicated cases as compliance is easier and the patient will need to abstain from alcohol for a shorter period of time. Intravaginal metronidazole gel is not recommended because it does not reach adequate concentrations in the vaginal vault. Intravaginal clindamycine gel has been shown to be not effective against trichomoniasis. Nitazoxanide is active in vitro against T. vaginalis, but associated with clinical failures and is therefore not a recommended therapy for trichomoniasis. In addition to treating the woman, all sexual partners should be referred for evaluation and receive presumptive therapy for trichomoniasis. Women treated for trichomoniasis should return to clinic for repeat testing within 3 months due to the high rate of reinfection. The retesting should not be done within 2 weeks of completing therapy as nucleic acid amplification testing may detect killed organisms and lead to unnecessary retreatment of the patient.

A 21-year-old woman, gravida 2, para 1, at 20 weeks of gestation, presents to your office and reports increasing gray vaginal discharge that has an odor. A wet mount reveals leukocytes and motile trichomonads. She denies pruritus. She does not have any significant medical history or allergies to medication. The next step in management of this patient is: A. Oral metronidazole, with partner treatment B. Vaginal metronidazole C. Oral azithromycin, with partner treatment D. Vaginal clindamycin, with partner treatment E. Oral fluconazole

A. Oral metronidazole, with partner treatment Explanation: Oral metronidazole is the method of treatment; vaginal therapy alone may be ineffective. Partner treatment is indicated to ensure cure, due to a 25% reinfection rate with an untreated partner. Vaginal metronidazole and clindamycin are effective treatments for bacterial vaginosis. Azithromycin may be used for treatment of chlamydia. Oral fluconazole is a treatment for candidal infections of the vulva and vagina.

Which of the following lower genital tract organisms is not associated with increased puerperal infection? A. Trichomonas vaginalis B. Group B streptococcus C. Gardnerella vaginalis D. Mycoplasma hominis E. Staphylococcus aureus

A. Trichomonas vaginalis Explanation: Most postpartum infections are caused by organisms that are present in the female genital tract and which also normally cause female genital tract infections. The common bacterial agents are include the following: • Gram-positive cocci: group A, B, and D streptococci, Staphylococcus aureus. • Gram-positive bacilli: Clostridium species. • Aerobic gram-negative bacilli: Escherichia coli, Klebsiella, Proteus species. • Anaerobic gram-negative bacilli: Bacteroides fragilis group. • Other: Mycoplasma species, Chlamydia trachomatis, Neisseria gonorrhoeae.

A 45-year-old woman presents to her local general practice nurse appointment for a routine smear test. She has not been for regular appointments with the nurse before. Whilst performing the smear test the nurse notices that the surface of the cervix is dotted with small haemorrhages and that there is an offensive, frothy, yellow-green discharge in the vagina with multiple erythematous areas on the vaginal walls. What is the most likely causative organism? A. Trichomonas vaginalis B. Chlamydia trachomatis C. Candida albicans D. Haemophilus ducreyi E. Bacterial vaginosis

A. Trichomonas vaginalis Explanation: TV is a sexually transmitted infection that causes inflammation of the vagina (vaginitis) and can often be confused with bacterial vaginosis (BV) a non-sexually transmitted infection that does not affect the vagina itself. The discharge is classically described as a 'champagne discharge' as it is green-yellow and frothy and may accompany a 'strawberry cervix' - so-called as the cervix is often dotted with small haemorrhagic areas. Diagnosis is with dark-ground microscopy on a wet prepar-ation and treatment involves metronidazole 400mg twice daily for a week.

A 36-year-old woman presents to the sexual health clinic with an offensive creamy yellow vaginal discharge. A saline wet mount of the discharge under microscope revealed motile, flagellated protozoa. Which is the most likely cause of this infection? A. Trichomonas vaginalis B. Neisseria gonorrhoeae C. Haemophilus ducreyi D. Treponema pallidum E. Herpes simplex

A. Trichomonas vaginalis Explanation: Trichomonas vaginalis (TV) is a single-celled, flagellated, motile protozoan. It is slightly larger than a granulocyte and depends on adherence to the host cell for its survival. Women can present with yellowish-green frothy vaginal discharge (has odour), itching of the genital area, dysuria and dyspareunia (vaginitis, cervicitis and urethritis). It may lead to premature rupture of membranes and preterm delivery. It can coexist with other genital infections such as gonorrhoea, Chlamydia and bacterial vaginosis. Most men are usually asymptomatic and can (rarely) develop genital irritation, epididymitis and prostatitis. On speculum examination, the vaginal mucosa is erythematous and the cervix is inflamed with numerous petechiae (strawberry appearance). Motile organisms are seen on wet-mount saline preparation under the microscope. Wet-mount microscopy and culture are the gold standard for its diagnosis. Metronidazole is the drug of choice for treatment.

A 22-year-old woman presents with intensely irritating yellowish-green frothy vaginal discharge with severe dyspareunia. The organism is seen best under microscope in a drop of saline. What is the most likely causative agent? A. Trichomonas vaginalis B. Gardnerella vaginalis C. Chlamydiae trachomatis D. Candida albicans E. Mycobacterium tuberculosis

A. Trichomonas vaginalis Explanation: Trichomonas vaginalis is a flagellated protozoan. It causes trichomoniasis, which is a sexually transmitted infection. Women usually present with vaginal discharge (70%) or frothy yellowish vaginal discharge (<30%), vulval itching and dysuria (urethral infection is seen in 90% of cases). Some women may present with lower abdominal pain and a few others may show signs of vaginitis and vulvitis. However, 50% of women may be asymptomatic. Contact bleeding from the cervix and strawberry appearance of the cervix (2% of cases) can be seen on speculum examination. A wet mount on microscopy will reveal a flagellated organism. A high vaginal swab for culture will be diagnostic in most cases (95%). The drug of choice for its treatment is metronidazole (single dose of 2 gm or 400 mg twice daily for 7 days) and this can be used in all trimesters of pregnancy. However, the use of a higher dose regimen should be avoided during pregnancy. Screening for other sexually transmitted diseases (STIs) should be offered and sexual partners should be treated. They should also be advised abstinence until treatment is completed.

A 35-year-old woman presented with a frothy vaginal discharge that is fishy in odour. On speculum examination, the cervix was red, punctate and inflamed. The wet mount shows the mobile organism. What is a likely diagnosis in her case? A. Trichomoniasis B. Candidiasis C. Bacterial vaginosis D. Chlamydia E. Donovanosis

A. Trichomoniasis Explanation: • Urethral infection is present in 90% of cases. • It is a sexually transmitted condition. • Approximately 50% of infected women are asymptomatic. • The most common symptoms include vaginal discharge (vaginits), vulval itching (vulvitis), dysuria or an offensive odour. • There may occasionally be lower abdominal pain. • Vaginal discharge is present in 70% of cases - classical frothy, yellow-green discharge and bad smelling. • The so-called 'strawberry cervix' with its characteristic vascular pattern is only present in 2% of cases, although this may be more visible at colposcopy. • Diagnosis rests on direct observation of the organism on a wet smear (mobile trichomonads are visible on the slide). • Specialised culture media are required and will be diagnostic in 95% of cases. • Trichomonads may occasionally be seen on cervical smears, though not all laboratories will report them because of a significant false-positive rate. It is prudent to confirm the diagnosis by direct observation or culture.

A 14-year-old adolescent female is here to see you for complaints of greenish vaginal discharge. She is sexually active with one partner and does not use condoms. You do a culture and find that she has Trichomonas vaginitis. She asks you not to tell her mother about this diagnosis or that she is sexually active. Which of the following statements is most accurate regarding disclosure or nondisclosure of this information to her parents? A. You can keep this information confidential. However, it is advisable to talk with the teen about her sexual history and discuss communication issues between her and her parents B. Inform her parents that she is engaging in unsafe sex C. Since she is a minor, you must disclose this information to her parents D. You can only keep this confidential for today for enhancing therapy, but then disclosure to the parents must be demonstrated and documented E. You may keep this confidential from the parents but you must call the partner to notify him of the infection

A. You can keep this information confidential. However, it is advisable to talk with the teen about her sexual history and discuss communication issues between her and her parents Explanation: The law does not require the disclosure of sensitive medical information to parents. However, in some states it is not forbidden to disclose that information. A clinician must use his or her best judgment when deciding whether to disclose medical information. More importantly, the physician should recognize the importance of confidentiality when treating patients and encourage open communication between adolescents and parents when it is reasonable to do so. Partner notification can occur by patient referral or by health department staff.

A 35-year-old woman presents with a malodorous, profuse, frothy, greenish vaginal discharge with intense vaginal itching and discomfort. Laboratory diagnosis of vaginal trichomoniasis is most commonly made by which of the following methods? A. Identifying cyst stages in an iodine stained preparation of vaginal secretion B. Finding trophozoites in a saline wet mount of vaginal discharge C. Using an acid-fast stain to highlight the parasite D. Staining a thin blood smear with common blood stains E. Testing for specific antibodies against T. vaginalis in the patient's serum

B. Finding trophozoites in a saline wet mount of vaginal discharge Explanation: Trophozoites are usually visible in saline mounts of vaginal discharge or scrapings. Cysts (A) are not present in the T. vaginalis life cycle, and iodine is used primarily to observe cysts of intestinal protozoa. Acid-fast stains (D) are used to search for oocysts of coccidian intestinal parasites, such as Cryptosporidium and Cyclospora. Thin blood smears (D) are used to diagnose malaria. Serologic diagnoses (E) are helpful in the diagnosis of several "deep tissue" parasites but are not used in diagnosing T. vaginalis.

A 17-year old female presents with profuse, greenish, bubbly vaginal discharge is noted to have red spots on the cervix. A wet mount indicates a Trichomonas vaginalis infection. you treat her with a single 2 g dose of oral metronidazole. The patient returns in 3 weeks with continued symptoms. She reports compliance with her initial treatment. Infection recurrence is confirmed on microscopy. Which of the following is the most appropriate treatment option? A. Tinidazole 2 g orally daily for 5 days B. Metronidazole 500 mg orally twice daily for 7 days C. Consultation with infectious disease specialist D. T. vaginalis susceptibility testing E. Clindamycin 300 mg orally twice times daily for 7 days

B. Metronidazole 500 mg orally twice daily for 7 days Explanation: Metronidazole 500 mg orally twice a day for 7 days is recommended for cases of metronidazole single-dose treatment failure. Treatment with tinidazole 2 g orally daily for 5 days is recommended after treatment failure with both the single-dose and the multidose metronidazole regimens. Consultation with infectious disease specialist is recommended only for refractory cases after multiple treatment failures. Reinfection and noncompliance should be ruled out. T. vaginalis susceptibility testing is available through the CDC. It is recommended only for refractory cases after multiple treatment failures. Reinfection and noncompliance should be ruled out. Clindamycin can be used as an alternative regimen for the treatment of bacterial vaginosis. Nitroimidazoles are the only class of drugs useful for the treatment of Trichomonas infections.

A 28-year-old African American woman comes into the sexually transmitted diseases clinic with her boyfriend to get an HIV test and to get screened for any sexually transmitted infections. She has a history of several STDs in the past. The routine screening ordered for this woman included screening for Trichomonas vaginalis infection. Which one of the following would you recommend as the preferred screening test for detecting T. vaginalis infection in this asymptomatic woman? A. Wet-mount microscopy of vaginal secretions B. Nucleic acid amplification testing (NAAT) on a vaginal swab specimen C. Culture of a first-catch urine specimen D. Culture of a midstream urine specimen E. Liquid based cytology of cervical smears

B. Nucleic acid amplification testing (NAAT) on a vaginal swab specimen Explanation: Routine screening for Trichomonas vaginalis infection in women is not routinely performed, but should be considered in persons receiving care in high prevalence settings, such as in sexually transmitted diseases clinics, and in women considered at high risk of having trichomoniasis (a history of sexually transmitted diseases, multiple sexual partners, illicit drug use, and exchanging sex for money). In addition, routine screening for T. vaginalis infection is recommended for women with HIV infection, primarily because of the adverse effects that can follow in women with asymptomatic infection, including an increased risk of developing pelvic inflammatory disease. A highly sensitive and specific test should be used for the diagnostic evaluation of T. vaginalis infection. In recent years, the T. vaginalis nucleic acid amplification test (NAAT) has emerged as a preferred screening and diagnostic test by most experts, primarily because of very high sensitivity and specificity (both greater than 95%). In women, the T. vaginalis NAAT performs very well on urine specimens and even better on vaginal and endocervical swabs (Figure 1). Wet mount microscopy is often used as part of the general evaluation of vaginal discharge, and although it does provide a convenient and rapid means to diagnose trichomoniasis, it has a sensitivity of only about 45 to 60%. If a specimen is collected for wet mount, the slide should be viewed promptly, since the sensitivity will decline over time, even within 1 hour. Prior to the availability of T. vaginalis NAAT, culture of vaginal secretions was the gold standard test with the highest sensitivity; using culture to diagnose trichomoniasis has fallen out of favor because the sensitivity of culture (75 to 96%) is lower than the sensitivity of the T. vaginalis NAAT. If culture is used, vaginal swab specimens are preferred over urine samples. Although T. vaginalis may be an incidental finding on a Pap test, neither conventional nor liquid-based Pap tests are considered diagnostic tests for trichomoniasis, because false negatives and false positives can occur.

An 18-year-old college student presents to the student health clinic with a complaint of copious yellow vaginal discharge. She has been sexually active with a new partner for the past month, but she is unsure if her partner is monogamous. A speculum examination reveals petechiae in the upper vagina and malodorous, yellow-green discharge. A potassium hydroxide preparation reveals no organisms, and results of a Giemsa stain are shown in the image. Which of the following is the most appropriate treatment? A. Treat her and her partner with oral fluconazole and test for other sexually transmitted diseases B. Treat her and her partner with oral metronidazole and test for other sexually transmitted diseases C. Treat her with oral fluconazole and test for other sexually transmitted diseases D. Treat her with oral metronidazole and test for other sexually transmitted diseases E. Treat her with vaginal fluconazole and test for other sexually transmitted diseases F. Treat her with vaginal metronidazole and test for other sexually transmitted diseases

B. Treat her and her partner with oral metronidazole and test for other sexually transmitted diseases Explanation: The wet prep revealed infection with the motile protozoan Trichomonas vaginalis. Typical signs and symptoms include a malodorous, green-yellow, frothy discharge, burning, pruritus, dysuria, frequency, dyspareunia, and punctate hemorrhages on the vagina/cervix. Metronidazole is the treatment for this infection. Oral treatment is preferred to vaginal treatment because it allows for therapeutic levels in the urethra and periurethral glands, which otherwise serve as a source for endogenous recurrence. This is an STD, so both she and her partner should be treated. Additionally, she should be tested for other STDs because infection with one STD carries a higher risk of harboring another STD. Answer A is incorrect. Oral fluconazole is used to treat fungal infections, including that of the mouth, throat, esophagus, lungs, and blood. Answer C is incorrect. Oral fluconazole is used to treat fungal infections, including that of the mouth, throat, esophagus, lungs, and blood. Answer D is incorrect. Treatment must also include treatment of the partner so the infection is not passed back and forth or to others. Answer E is incorrect. Vaginal fluconazole is used to treat Candida albicans infection of the vagina, not Trichomonas infection. Answer F is incorrect. Oral treatment is superior to vaginal treatment, so the urethra can be treated to prevent self-reinfection. Treatment must also include treatment of the partner and testing for other sexually transmitted diseases (STDs).

Flagellate parasite is the cause of vulvovaginitis in women in temperate areas. It is a sexually transmitted disease. The prominent symptom is copious fowl-smelling yellow discharge that can be accompanied by cervical lesions and abdominal pain. What parasite does this represent? A. Entamoeba histolytica B. Trichomonas vaginalis C. Strongyloides stercoralis D. Schistosoma mansoni E. Balantidium coli

B. Trichomonas vaginalis Explanation: This is a case of Trichomonas vaginalis infection showing a profuse discharge stemming from the cervical os. Trichomonas vaginalis is an important sexually transmitted flagellate. An estimated 250 million new cases of Trichomoniasis occur around the world with nine million new cases in the United States every year. The parasite inhabits predominately the female lower genital tract and can also be found in the male urethra and prostate. The parasite divided by binary fission and does not have a cyst form and is transmitted human to human primarily by sexual intercourse. T. vaginalis infection in women is frequently symptomatic. Vulvovaginitis with a profuse fowl-smelling yellow discharge is the prominent symptom and can be a ccompanied by cervical lesions and abdominal pain. Growth of the organism is favored by high pH >5.9 (normal pH in vagina = 3.5-4.5). In males, the infection is often asymptomatic but urethritis, epididymitis, and prostatitis can occur.

Which of the following organisms may be isolated from a wet surface 6 hours after inoculation? A. Candida albicans B. Trichomonas vaginalis C. Gardenerella species D. Mobiluncus species E. Peptostreptococci

B. Trichomonas vaginalis Explanation: Trichomonas vaginalis is a hardy organism and may be isolated from a wet surface up to 6 hours after inoculation. Its difficulty to kill is the reason that therapy is metronidazole 2 gm orally all at once, to be able to obtain sufficiently high tissue levels to be effective. Not uncommonly, a single course is not effective, and a 2- or 3day course of metronidazole a high dose orally is needed.

A 28-year-old female presents with two-month history of thick bubbly discharge and intense itching. Which of the following is the most likely cause of her discharge? A. Chlamydiae trachomatis B. Trichomonas vaginalis C. Candida albicans D. Sarcoptes scabeii E. Gardrenella vaginalis

B. Trichomonas vaginalis Explanation: Trichomonas vaginitis is a sexual transmitted infection caused by a flagellated protozoan that can reside asymptomatically in male seminal fluid. The main patient complaint is vaginal discharge associated with itching, burning, and pain with intercourse. On speculum examination, vaginal discharge is typically frothy and yellowish. The vaginal epithelium is frequently edematous and inflamed. The erythematous cervix may demonstrate the characteristic 'strawberry' appearance. Vaginal ph is elevated > 4.5. Wet mount microscopic examination reveals actively motile 'trichomonads' on a saline preparation. WBCs are seen. The treatment of choice is oral metronidazole for both the patient and her sexual partner. Vaginal metronidazole gel has a 50% failure rate. Metronidazole is safe to use during pregnancy, including the first trimester.

A 20-year-old woman attends the sexual health clinic with symptoms of frothy, yellow vaginal discharge and associated lower abdominal pain. The organism can be seen when a drop of saline is added to the vaginal discharge placed on the slide. What is the most likely diagnosis? A. Chlamydia B. Trichomonas vaginalis C. Gonorrhoea D. Candida albicans E. Syphilis

B. Trichomonas vaginalis Explanation: Trichomoniasis is a sexually transmitted disease caused by the parasite Trichomonas vaginalis. Symptoms usually appear 5-28 days after exposure and include a yellow- green vaginal discharge, often foamy, with a strong odour, dyspareunia and vaginal irritation. Two per cent of women infected also manifest a 'strawberry' cervix on examination. T. vaginalis is a flagellated organism which can damage the vaginal epithelium, increasing a woman's susceptibility to infection by human immunodeficiency virus (HIV). This is caused by lysis of the epithelial cells. Treatment is with metronidazole 400 mg twice daily for 7 days or tinidazole at 2 g as a single dose. As this is a sexually transmitted disease, diagnosis should prompt the gynaecologist to refer the patient to a genitourinary medicine clinic for contact tracing.

A female patient presents with reddish vagina and greenish discharge with severe itching points, the probable diagnosis is: A. Herpes infection B. Trichomonasis C. Gonococcal infection D. Candidiasis E. Pyogenic vaginitis

B. Trichomonasis Explanation: Trichomonasis is a STI caused by trichomonas vaginalis —flagellated protozoan. Nearly 5000 reported cases in England in 2010. Trichomonas vaginalis may be found in vaginal, urethral, and para-urethral glands. Asymptomatic in 10-50%, but may present with: • Frothy, greenish, offensive smelling vaginal discharge. • Vulval itching and soreness. • Dysuria. • Cervix may have a 'strawberry' appearance from punctate haemorrhages (2%). Diagnosis is made by direct observation of the organism by a wet smear (normal saline) or acridine orange stained slide from the posterior vaginal fornix (sensitivity 60-70% cases). Culture media are available and will diagnose up to 95% cases. NAATs have been developed and sensitivities and specificities approaching 100% have been reported. Treatment of choice is metronidazole 2g orally in a single dose or metronidazole 400-500mg BD for 5-7 days. Contact tracing and treatment of sexual partners is recommended. There is some evidence that trichomonal infection may enhance HIV transmission. Trichomonas in pregnancy is associated with: • Preterm delivery. • Low birth weight. • Trichomonas may be acquired perinatally, occurring in 5% of babies born to infected mothers.

You are asked to see a 33 year old women who is 16 weeks pregnant. She complains of frothy yellow vaginal discharge and vaginal soreness. A wet smear is sent and microscopy reveals Trichomoniasis. What percentage of women with trichomoniasis infection have the classic frothy yellow PV discharge? A. 2% B. 5% C. 20% D. 50% E. 70%

C. 20% Explanation: Trichomoniasis Key Points: • Trichomoniasis is a sexually transmitted infection by the flagellate protazoan Trichomonias Vaginalis. • 10-50% of women have no symptoms and 5-15% of women have a normal examination. • Vaginal discharge is most common symptom in (up to 70%). Classic discharge described as frothy and yellow-green only occurs in (10-30% of infected women) but can be variable. Other symptoms are vulvovaginal soreness and itching, offensive odour, lower abdominal pain, dysuria and dyspareunia. • Clinical findings are typically vulval inflammation and excoriation. Rarely strawberry cervix (cervicitis - 2% of cases). • Diagnosis wet smear microscopy or culture/PCR. • Metronidazole 400 to 500 mg twice a day for 5 to 7 days is 1st line treatment for men and women (including those who are breastfeeding or pregnant).

A 24-year-old woman is diagnosed with vaginal trichomoniasis. She is prescribed metronidazole. She is breastfeeding her 3 month old boy. What is the appropriate advice for this patient concerning the use of metronidazole during breastfeeding? A. Stop breast feeding and replace it with formula for 3 months B. Stop metronidazole and choose another less effective drug as you weigh the benefits and the risks C. Advise her to use the pump to collect her breast milk every 24 hours right before she takes daily metronidazole D. Advise her to keep breastfeeding her child as there is no risk of the drug on breast feeding E. Stop breast feeding indefinitely

C. Advise her to use the pump to collect her breast milk every 24 hours right before she takes daily metronidazole Explanation: The best advice for this patient is to use metronidazole once a day and collect breast milk by pump right before she takes metronidazole. All other choices are inappropriate for this patient.

Protozoans exhibit diverse modes of locomotion across the various groups. The motility of trichomonas vaginalis is described as A. Amoeboid B. Jerky C. Falling leaf like D. Stately E. Ciliary

C. Falling leaf like Explanation: Unlike other parasitic protozoa, trichomonas vaginalis exists in only one morphological stage, a trophozoite, and cannot encyst. The T. vaginalis trophozoite is oval as well as flagellated, or "pear" shaped as seen on a wet-mount. It is slightly larger than a white blood cell, measuring 9 × 7 μm. Five flagella arise near the cytostome; four of these immediately extend outside the cell together, while the fifth flagellum wraps backwards along the surface of the organism. The functionality of the fifth flagellum is not known. In addition, a conspicuous barb-like axostyle projects opposite the four-flagella bundle. The axostyle may be used for attachment to surfaces and may also cause the tissue damage seen in trichomoniasis infections. While T. vaginalis does not have a cyst form, organisms can survive for up to 24 hours in urine, semen, or even water samples.

A 45-year-old sex worker is seen at the local health department with complaint of a malodorous vaginal discharge for 2 weeks. After thorough examination and wet prep you treat her with metronidazole. What did the wet prep show? A. Rods and buds B. Clue cells C. Flagellated organisms D. Parabasal cells E. Fat globules

C. Flagellated organisms Explanation: Trichomonads are unicellular, flagellated, anaerobic organisms. They cause sexually transmitted vaginitis characterized by abnormal vaginal odor and pruritus of the vulva and vagina. The classic sign of Trichomonas is a "strawberry cervix;' though this finding is only seen in about 2 % of cases. Metronidazole is the treatment of choice.

A 35-year-old woman complains of itching in the vulval area. Hanging-drop examination of the urine reveals trichomonads. What is the preferred treatment for the trichomoniasis? A. Doxycycline B. Emetine C. Metronidazole D. Pentamidine E. Pyrazinamide

C. Metronidazole Explanation: Metronidazole is distributed to all body fluids and tissues. Its spectrum of activity is limited largely to anaerobic bacteria—including B. fragilis—and certain protozoa. It is considered to be the drug of choice for trichomoniasis in females and carrier states in males, as well as for intestinal infections with Giardia lamblia. Doxycycline (a), emetine (b, an antiprotozoal), pentamidine (d; used for treatment of P. jirovecii, pneumonia, or prophylactically for that condition in high-risk HIV-positive patients), and pyrazinamide (e, a bactericidal agent used for M. tuberculosis infections) would not be appropriate for the patient described in the question.

A woman has a sexually transmitted disease, and the decision is made to treat her with antibiotics as an outpatient. She is warned that unpleasant reactions may occur if she consumes alcoholic beverages while taking this drug. The antibiotic can be identified as which of the following? A. Ceftriaxone B. Doxycycline C. Metronidazole D. Ofloxacin E. Penicillin G

C. Metronidazole Explanation: Organisms associated with sexually transmitted diseases include chlamydia, neisseria gonorrhea, treponema (syphilis), trichomonas, and gardnerella vaginalis. The latter two organisms are effectively treated with the drug metronidazole. Metronidazole has a chemical structure that results in a disulfiram-like effect on aldehyde dehydrogenase, causing reactions with ethanol. Patients should be cautioned not to consume alcoholic beverages while on this drug.

A 17-year old female is presenting with intense vaginal pruritus and motile flagellated organisms on wet prep. She is diagnosed with trichomonas. What is the treatment of choice? A. Benzylpenicillin B. Erythromycin C. Metronidazole D. Fluconazole E. Trimethoprim

C. Metronidazole Explanation: Metronidazole (Flagyl) a prodrug, is a bactericidal agent that is primarily used because of its anaerobic antiprotozoal activity, particularly in the treatment of trichomoniasis. It is also employed as an antibacterial in treating certain anaerobic bacterial infections. It has been proposed that an intermediate in the reduction of metronidazole, produced only in anaerobic bacteria and protozoa, is bound to DNA and electron-transport proteins, thus inhibiting nucleic acid synthesis. Patients using metronidazole should avoid alcoholic beverages to avoid disulfiram-like effects when the combination is used. Neuropathy is seen with long-term use.

A 25-year-old G2P2 female complains of a large amount of foul-smelling vaginal discharge. On physical exam, you notice a frothy, yellow-green discharge and multiple petechiae on the cervix. The wet mount of the discharge shows motile protozoa. What is the treatment of choice for this patient? A. Ceftriaxon B. Doxycycline C. Metronidazole D. Azithromycin E. Clindamycin

C. Metronidazole Explanation: Metronidazole is the treatment of choice for trichomoniasis. In addition to the classic frothy, yellow-green malodorous discharge, petechiae are often seen on the cervix during exam (commonly called strawberry cervix).

An 18-year-old G2P2 woman presents to her gynecologist's office complaining of several days of vaginal itching and discomfort on urination. She denies any recent fever, back pain, hematuria, or vaginal bleeding. She has been sexually active with one partner and uses barrier protection "most of the time." A urine β-human chorionic gonadotropin screen is negative. On examination she has a moderate amount of yellow discharge. Results of Giemsa staining of McCoy cells are shown in the image. Which of the following is the most effective oral pharmacotherapy for this disorder? A. Azithromycin B. Fluconazole C. Metronidazole D. Penicillin E. Vancomycin

C. Metronidazole Explanation: The constellation of symptoms and the presence of Trichomonas vaginalis protozoa on Giemsa staining establish the diagnosis of trichomoniasis vaginitis. The treatment of choice for this condition is one 2-mg dose of metronidazole. Answer A is incorrect. Azithromycin is a macrolide antibiotic that inhibits RNA synthesis by binding to the 50S ribosomal RNA subunit. It is used to treat chlamydial infections but is not therapeutic for trichomoniasis. Answer B is incorrect. Fluconazole is an antifungal medication that inhibits cell membrane synthesis of various fungi. It is often used to treat vaginal candidiasis, but is has no antiprotozoal activity and thus is ineffective against Trichomonas vaginalis infections. Answer D is incorrect. Penicillin is an antibiotic whose mechanism of action involves interruption of cell wall synthesis. Its spectrum of activity includes most grampositive cocci, most anaerobes, and a limited number of gram-negative bacteria. It is not active against Trichomonas vaginalis. Answer E is incorrect. Vancomycin is an antibiotic that is effective against numerous gram positive organisms, including methicillin resistant Staphylococcus aureus, and which is almost always administered as an intravenous infusion. Vancomycin is not active against Trichomonas vaginalis.

You are seeing a 17-year-old girl who reports intense vaginal itching and urinary frequency. She has been sexually active for 6 months. On examination, you note frothy yellow-green discharge with bright red vaginal mucosa and red macules on the cervix. What is the saline preparation of the discharge most likely to show? A. Sheets of epithelial cells "studded" with bacteria B. "Moth-eaten" epithelial cells C. Motile triangular organisms with long tails D. Numerous white blood cells E. Hyphae

C. Motile triangular organisms with long tails Explanation: The history and physical described are classic for trichomonas vaginalis. The classic "strawberry cervix" is a strong diagnostic clue. Trichomonads are seen on high power in the saline preparation, and appear as triangular cells with long tails, slightly larger than WBC. "Studded" epithelial cells (clue cells) are more consistent with bacterial vaginosis; "moth-eaten" cells (pseudo-clue cells) are seen in an acid-base disturbance of the vagina. Numerous WBC are more consistent with an upper genital infection, and hyphae are consistent with vaginal candidiasis.

A 15-year-old girl presents with a history of vaginal discharge, and Trichomonas is isolated from the vaginal swab. What is the most appropriate antibiotic for this infection? A. IM ceftriaxone B. Oral erythromycin C. Oral metronidazole D. Oral fluoconazole E. Topical nystatin

C. Oral metronidazole Explanation: Trichomonas in young children raises the possibility of sexual contact and one should not rule out sexual abuse. T. vaginalis is an anaerobic protozoan resides not only in the vagina but also in the urethra, bladder, and Skene glands, so systemic, rather than local, therapy is needed. Metronidazole is effective in treating trichomoniasis; the preferred regimen is 2 g in one dose because of complete patient compliance and high effectiveness. Extended 7-day metronidazole therapy, 500 mg twice daily, does not increase the 95% cure rate of a single dose. Simultaneous treatment of the male sexual partner is recommended.

A patient states that she has a malodorous discharge and intense itching. She adds that her partner also has a slight discharge. Pelvic examination reveals "strawberry spots" on the cervix. What is the most appropriate initial management? A. Oral fluconazole B. Candid pessary C. Oral metronidazole D. Sulfonamide vaginal cream E. Vinegar douching

C. Oral metronidazole Explanation: Trichomoniasis is an STI caused by the unicellular protozoan Trichomonas vaginalis. Trichomonads can survive on wet towels and other surfaces and thus can be nonsexually transmitted. Its incubation period ranges from 4 to 28 days. Vaginal exam may reveal a frothy, malodorous yellow-green discharge with vulvar irritation and the cervix may appear erythematous and friable. However, many women have minimal or no symptoms. A wet smear preparation that is promptly reviewed may reveal the flagellated, mobile protozoon with a sensitivity of approximately 60% to 70%. Trichomonas culture tests have 95% sensitivity. Liquid-based testing Pap tests have increased sensitivity for T. vaginalis; however, false positives have occurred and confirmatory testing may be needed. More recent PCR-based diagnostic tests have sensitivities and specificities approaching 100%. Treatment consists of one 2-g dose of either metronidazole or tinidazole PO. Alternatively, metronidazole 500 mg PO bid for 7 days can be used. Metronidazole gel has an efficacy of <50% and is not recommended. Patients with allergy to metronidazole should be referred for desensitization and subsequent treatment with metronidazole. The patient's sexual partners should be treated as well.

A healthy 23-year-old woman presents to the clinic for reevaluation of dysuria and vaginal discharge. She was seen 3 weeks prior for a similar complaint. She engages in unprotected oral and vaginal sex with a single male partner. At the prior visit nucleic acid amplification testing (NAAT) of her urine was negative for Neisseria gonorrhea and Chlamydia trachomatis, but positive for Trichomonas vaginalis. She was treated with a single dose of 2 grams of metronidazole orally; her male sexual partner did not follow-up and get treated because he did not have any symptoms. At today's visit she undergoes repeat testing for sexually transmitted diseases and she again tests negative for C. trachomatis and N. gonorrhea, but positive for T. vaginalis. In addition, a urine culture is negative. She has had sex with her partner several times since completing treatment, but he has remained asymptomatic. At this point, which one of the following represents the best option for the management of this situation? A. Prescribe the patient tinidazole 2 grams orally in a single dose B. Prescribe the patient metronidazole 250 mg orally three times a day plus doxycycline 100 mg twice a day for 7 days C. Prescribe the patient metronidazole 2 g orally in a single dose and ensure that her partner is referred and presumptively treated with metronidazole 2 g orally in a single dose D. Prescribe the patient metronidazole 2 g orally daily for 7 days E. Prescribe the patient tinidazole 2 g orally twice daily for 14 days

C. Prescribe the patient metronidazole 2 g orally in a single dose and ensure that her partner is referred and presumptively treated with metronidazole 2 g orally in a single dose Explanation: This patient is likely presenting with recurrent Trichomonas vaginalis infection. The most likely causes for persistent symptoms in patients following treatment for trichomoniasis include reinfection with T. vaginalis from an untreated partner, infection with another sexually transmitted pathogen, or T. vaginalis antimicrobial resistance. For most patients with recurrent infection with T. vaginalis, the most common cause is reinfection from an untreated sexual partner. Accordingly, the clinician should ensure treatment of all sexual partners. Specifically, the male partner should promptly be referred for presumptive therapy for trichomoniasis. For trichomoniasis, use of expedited partner therapy (providing prescription or medication to the patient to take to the partner) has not been established to reduce rates of reinfection. It is important to note that T. vaginalis infection is most often asymptomatic in men and the absence of symptoms should not preclude treatment of the male partner. Asymptomatic men may need counseling and an explanation for why they need treatment of trichomoniasis. Although T. vaginalis antimicrobial resistance is uncommon (4 to 10% with metronidazole and less than 1% with tinidazole), clinicians should be aware of the next steps when resistance is suspected. If reinfection is thought to be unlikely, treatment of the patient (and the sexual partners) should consist of a 7-day course of oral metronidazole 500 mg orally twice a day; single dose therapy with metronidazole or tinidazole should be avoided in this situation. If retreatment fails (and reinfection excluded), then the next option is to treat with a 7-day oral course with either metronidazole 2 g once daily or tinidazole 2 g once daily; if this approach fails, expert consultation for evaluation and treatment of nitroimidazole-resistant T. vaginalis resistance should be obtained through the Centers for Disease Control and Prevention (telephone 404-718-4141).

A 14-year-old girl presents to her family practitioner with a 2-day history of profuse vaginal discharge and pruritus. She admits that she has recently become sexually active with her boyfriend. The physician examines a potassium hydroxide mount of the vaginal discharge and sees many WBCs, but no hyphae. The pH of the discharge is higher than normal. What is the physician obligated to do next? A. Inform the patient's mother that she is engaging in unsafe sex B. Recommend abstaining from sexual intercourse for 2 weeks C. Recommend treatment of her boyfriend D. Report the case to the county department of health E. Screen the patient for diabetes

C. Recommend treatment of her boyfriend Explanation: This patient likely has Trichomonas infection. This is distinguished from candidiasis by the lack of hyphae on the potassium hydroxide wet mount and increased pH of the vaginal fluid. The disease classically "bounces" between partners as they reinfect each other, making treatment of the partner simultaneously an important aspect of management. Answer A is incorrect. As a minor seeing her physician because of an STD, the patient has the right to confidentiality. However, fully educating the patient about effective strategies for STD and pregnancy prevention, including abstinence, would be appropriate. Answer B is incorrect. This is not necessary for effective treatment. Answer D is incorrect. Reportable STDs are chlamydia, gonorrhea, and syphilis. The patient may have one of these conditions and should be tested for them, but Trichomonas is not necessary to report per se. Answer E is incorrect. Candida infection can be a presenting symptom of diabetes. However, the lack of hyphae in the vaginal discharge makes this diagnosis unlikely.

A 22-year-old woman comes to the physician with her husband because of vaginal irritation and a malodorous vaginal discharge. Her symptoms started 4 days ago. She also notes pain with intercourse and dysuria. Pelvic examination reveals vaginal and cervical erythema and a copious greenish, frothy discharge. The pH of this discharge is 6.0. A wet preparation is done with normal saline, which shows numerous flagellated organisms that are slightly larger than the surrounding white blood cells. Which of the following is the most appropriate management? A. Do not treat the patient or her partner B. Treat only the patient with metronidazole C. Treat the patient and her partner with metronidazole D. Treat only the patient with penicillin E. Treat the patient and her partner with penicillin

C. Treat the patient and her partner with metronidazole Explanation: Trichomoniasis, a sexually transmitted disease, is a common cause of vulvovaginitis. It is caused by the organism Trichomonas vaginalis. The most common symptoms are a profuse, malodorous, frothy vaginal discharge and vaginal itch and pain. Occasionally patients will have dysuria, dyspareunia, and pelvic pain. Examination will demonstrate the copious, frothy discharge. The pH of this discharge will usually be equal to or greater than 6.0. On a normal saline smear, motile trichomonads will be seen in most symptomatic patients. The drug of choice to treat trichomoniasis is metronidazole, and it is imperative that both the patient and her partner be treated. They should refrain from intercourse until they have completed the treatment and are asymptomatic. Failing to treat the patient or her partner (choice A) would not be an appropriate course of action. The patient has obvious trichomoniasis and therefore needs to be treated. Her partner also must be treated so that he does not continue to reinfect the patient if he is infected as well. Treating only the patient with metronidazole (choice B) would leave her partner untreated. If he is infected and does not receive treatment, he may continue to infect her. Treating only the patient (choice D) or the patient and her partner (choice E) with penicillin would not be appropriate. Penicillin is not a drug of choice for trichomoniasis. The first-line treatment is metronidazole.

An 18-year-old nulligravid female complains of a large amount of foul-smelling vaginal discharge. She is sexually active and monogamous with her boyfriend. They use condoms inconsistently. On physical exam, you notice a frothy, yellow-green discharge and multiple petechiae on the cervix. You diagnose her with trichomoniasis and prescribe her a single 2 g dose of oral metronidazole. What is another important component of the treatment plan? A. Performing a test of cure in 3 months B. T. vaginalis susceptibility testing C. Treatment of sexual partner(s) D. Empiric treatment for chlamydial infection E. No further action is needed

C. Treatment of sexual partner(s) Explanation: Because of the high rate of concomitant Trichomonas infections in sexual partners, all partners should be treated to prevent reinfection. No data supports the rescreening for T. vaginalis at 3 months following initial infection. This is likely due to the high efficacy of nitroimidazole drugs, with a 90% to 95% cure rate. T. vaginalis susceptibility testing is available through the CDC. It is only recommended for refractory cases after multiple treatment failures. Reinfection and noncompliance should be ruled out. Empiric treatment for chlamydial infection is not recommended with the diagnosis of Trichomonas infections. On the other hand, empiric treatment of chlamydial infection is recommended in patients infected with N. gonorrhoeae due to the high frequency of coinfection (unless chlamydial coinfection has been ruled out by nucleic acid amplification testing). Sexual partners should receive treatment.

A 25-year-old woman presents to her GP following an episode of unprotected intercourse. She complains of a foul smelling yellow vaginal discharge and dysuria. On examination you note evidence of a strawberry-red cervix. What is the most likely diagnosis? A. Chlamydia B. Genital herpes C. Trichomonas D. Neisseria gonorrhoeae E. Syphilis

C. Trichomonas Explanation: Trichomonas is a flagellate protozoon that infects the vagina, urethra and paraurethral glands. Ninety per cent of women with vaginal infection also have urethral infection. Up to 50% of women are asymptomatic, but common symptoms are an offensive odour, itching, profuse discharge and dysuria. Clinical findings are vulval excoriation, vulvovaginal inflammation, a profuse frothy yellow offensive discharge and 'strawberry cervix' (red and inflamed). Diagnosis is made by high vaginal swab, with direct observation of flagellates on a wet smear at microscopy. Alternatively, swab culture detects 95% of Trichomonas infections. The diagnosis is sometimes made on midstream specimen of urine microscopy or as an incidental finding on cervical smear. Twenty per cent of infections spontaneously resolve without treatment. However, treatment should always be given if the diagnosis is made by Metronidazole 2 grams orally as a single dose or Metronidazole 400 mg twice daily for 5 days. If metronidazole is ineffective, a single dose of tinidazole 2 grams orally may be used. Metronidazole and tinidazole high-dose regimes are not recommended at all in pregnancy, though there is no clear evidence of harm. Pregnant women should generally be treated with the 5-day metronidazole course. A test of cure is only needed if symptoms persist. Preterm delivery and low birth weight are both associated with maternal Trichomonas infection. Neonatal Trichomonas occurs in 5% of babies of untreated mothers.

What species of protozoa causes a foul-smelling vaginal discharge with itching and burning and on physical examination shows a fiery red cervix? A. Entamoeba B. Trypanosoma C. Trichomonas D. Leishmania E. Plasmodium

C. Trichomonas Explanation: Classification of medically important protozoan species causing disease in humans: A. Amoebae (sarcodina) subphyla Defining characteristics : Amoeboid movement with pseudopods. Medically important species :Entamoeba histolytica causes Amoebiasis (amoebic dysentery). B. Flagellates (mastigophora) subphyla Defining characteristics : Flagella that produces a whiplike movement. Medically important species : 1. Giardia lamblia causes Giardiasis. 2. Trichomonas vaginalis causes Trichomonal vaginitis. 3. Leishmania spp. causes Leishmaniasis. 4. Trypanosoma spp. causes Trypanosomiasis (sleeping sickness and Chagas' disease). C. Cilates (ciliophora) subphyla Defining characteristics : Cilia beat to produce movement. Medically important species : Balantidium coli causes diarrhea. D. Sporozoans (sporozoa) subphyla Defining characteristics : No locomotor organs in adult stage. Medically important species : Plasmodium spp. causes Malaria.

A 19-year-old woman presents to the physician's office for the evaluation of an itchy vaginal discharge that she has had for about a week. She has had no fever, abdominal pain, or dysuria. She became involved with a new sexual partner approximately 3 weeks ago. She takes birth control pills but does not regularly use condoms during intercourse. Her partner is asymptomatic. On examination, her vital signs are normal, and a general physical examination is unremarkable. On pelvic examination, her external genitalia are normal. After inserting a speculum you see a bubbly, thin, yellow vaginal discharge. Her cervix is erythematous but without discharge. She has no cervical motion or uterine or adnexal tenderness. A wet mount of the vaginal discharge examined microscopically reveals numerous motile, flagellated, pear-shaped organisms along with numerous white blood cells. What is the most likely infectious cause of her vaginal discharge? A. Treponema pallidum B. Neisseria gonorrhoeae C. Trichomonas vaginalis D. Gardnerella vaginalis E. Haemophilus ducreyi

C. Trichomonas vaginalis Explanation: Trichomonas vaginalis is a motile, pear-shaped protozoan with four flagella and an undulating membrane. It multiplies by binary fission and exists only in its trophozoite form; no cyst form has been identified. It is a common cause of both symptomatic and asymptomatic infections. Many infected women are asymptomatic or have only a small amount of thin vaginal discharge. Others develop symptomatic disease with vaginal inflammation, itching, and copious vaginal discharge. The discharge may be white, yellow, or green, and bubbles are often seen. Cervical inflammation with punctate hemorrhages may produce a "strawberry cervix." The vast majority of infections in men are asymptomatic, although urethritis, prostatitis, and epididymitis can occur. The parasite is almost always passed by sexual contact, although fomite transmission has been documented. The diagnosis is most often made by the microscopic evaluation of a sample of vaginal discharge in a saline wet mount. Flagellated, motile trichomonads will be visible in most symptomatic infections. The diagnosis can also be made by the identification of organisms on Pap smears, by culture of the vaginal discharge, or by the use of specific monoclonal antibody stains or nucleic acid probes. This infection is usually treated with oral metronidazole, and both partners should be treated to prevent reinfection. Because of its route of transmission, the identification of infection with Trichomonas should prompt the consideration of evaluation for other sexually transmitted diseases.

A 25-yea-old female presents with yellow-green, pruritic, "frothy" vaginal discharge; On examination she has a marked erythemetous cervix. A normal saline wet preparation is performed that demonstrates motile and flagellated organisms. Which of the following is the most likely causal organism? A. Treponema pallidum B. Neisseria gonorrhoeae C. Trichomonas vaginalis D. Gardnerella vaginalis E. Haemophilus ducreyi

C. Trichomonas vaginalis Explanation: Trichomoniasis is caused by Trichomonas vaginalis. Syphilis is caused by Treponema pallidum. Syphilis is divided into different stages. Primary syphilis is characterized by a nontender, red, round, firm ulcer approximately 1 cm in size with raised edges, known as chancre. None of these findings were presents in this patient. Neisseria gonorrhoeae causes gonorrhea. Approximately 50% of women with gonorrhea are asymptomatic. N. gonorrhoeae is a common cause of mucopurulent cervicitis and pelvic inflammatory disease. This patient does not have clinical findings of either of these conditions. Bacterial vaginosis is typically caused by Gardnerella vaginalis. Haemophilus ducreyi causes chancroid, which is an ulcerated genital lesion. This patient did not present with an ulcerated lesion.

A multiparous woman of 16 weeks gestation reports an intensely itchy, greencoloured offensive vaginal discharge. Which of the following is the most likely cause of her discharge? A. Gonorrhoea B. Candidiasis C. Trichomoniasis D. Listeriosis E. Toxoplasmosis

C. Trichomoniasis Explanation: Trichomoniasis is an STI in which the male is transiently colonized from the reservoir in the female vagina. This occurs because the adhesion proteins of this protozoon organism are expressed preferentially in acid pH, typically found in the vagina whereas the male urethral pH is approximately 7.4. In addition, trichomonads are repeatedly washed out of the urethra when men micturate. The organism can colonize the male prostate, and the female equivalent (Skene's glands). The woman has an intensely irritating vaginal discharge with inflammation of the vulva, vagina and cervix. The cervical appearance may manifest as a 'strawberry cervix'. The vaginal discharge is greenish-yellow, frothy and offensive with a fish like smell. Men typically have no symptoms but, when present, the symptoms include urethral discomfort, dysuria and occasional urethral discharge. Diagnosis may be made with the use of a saline wet-mount in which a drop of vaginal fluid from the posterior fornix is placed in a drop of saline on which a coverslip is floated. The trichomonads have a characteristic motility due to their five flagella. Culture is more sensitive although the gold standard is DNA amplification, currently unavailable in many clinical settings. It is important to screen for other common STIs. This is an STI, so sexual partner(s) must be treated with antimicrobials. Recommended regimen is metronidazole 2g single dose or 400mg twice a day for five days. Alternative regimen is tinidazole 2 g single dose. Various complicated regimens exist for metronidazole resistant trichomoniasis and referral to an STI specialist is recommended. Partner notification (contact tracing) should be initiated at diagnosis. Patients should return for a test of cure when they remain symptomatic following treatment, or if symptoms recur. Vaginal infections in pregnancy are associated with an increased risk of premature rupture of membranes, preterm delivery, and low birth weight.

A 24-year-old woman recently diagnosed with HIV infection presents for her initial visit with her new primary care provider. She acquired HIV through unprotected vaginal sex with a male partner. She is feeling well and without specific complaint at today's visit. As part of her visit, you are planning to do routine screening for sexually transmitted infections. Which one of the following is TRUE regarding Trichomonas vaginalis infection in women with HIV infection? A. Trichomoniasis screening is not recommended since women with HIV infection have low prevalence rates when compared to women without HIV infection B. Trichomoniasis is associated with lower vaginal and endocervical HIV RNA levels and reduced HIV genital tract shedding C. Trichomoniasis is associated with an increased incidence of pelvic inflammatory disease, increased genital tract HIV RNA levels, and enhanced HIV shedding D. The recommended treatment for trichomoniasis is the same in women with HIV infection as in women without HIV infection E. Trichomoniasis retesting is not recommended In women with HIV infection who receive initial treatment

C. Trichomoniasis is associated with an increased incidence of pelvic inflammatory disease, increased genital tract HIV RNA levels, and enhanced HIV shedding Explanation: Trichomoniasis occurs in up to 53% of women with HIV infection and is associated with an increased incidence of pelvic inflammatory disease, higher genital tract HIV RNA levels, and enhanced HIV shedding. Thus, routine screening for T. vaginalis infection is recommended at entry into care and thereafter at least annually. Prompt treatment of T. vaginalis is recommended in all women with HIV infection with evidence of trichomoniasis. Results from a randomized clinical trial involving women with HIV infection who had trichomoniasis showed a higher failure rate with a single-dose 2-gram metronidazole therapy than with a 7-day course of 500 mg metronidazole given twice daily (Figure 1). Based on results from this study, single-dose metronidazole therapy is not recommended for the treatment of trichomoniasis in women with HIV infection.

Vulvovaginal trichomonas can be described most accurately by which of the following statements? A. Caused by the overgrowth of bacteria native to some individuals B. Not associated with an alteration of normal vaginal pH C. Causes pelvic inflammatory disease D. A sexually transmitted disease E. Diagnosis can be made by obtaining a wet smear and mixing with KOH (potassium hydroxide)

D. A sexually transmitted disease Explanation: Trichomonas is in most cases a sexually transmitted disease caused by a protozoan, not bacteria. It is generally more common when the pH has become more alkaline than the normal vagina (postmenstruation, vaginosis, etc.). It may often be asymptomatic. Successful eradication requires treatment of all sexual partners. Trichomonads can be identified as pear-shaped motile organisms when a sample of vaginal discharge is mixed with normal saline. A KOH (potassium hydroxide) preparation is useful for identifying the hyphae associated with candidal vaginitis.

A female who is pregnant is diagnosed to have trichomonas vaginalis infection. What is not true regarding pregnancy complications and trichomonas? A. Premature rupture of membranes (PROM) is associated with trichomonas B. Low birthweight (LBW) and preterm delivery are associated with trichomonas C. Treatment of trichomonas has not been shown to decrease pregnancy complications in studies D. All pregnant women are routinely screened for trichomonas E. Although perinatal transmission of trichomoniasis is uncommon, treatment might prevent respiratory or genital infection of the newborn

D. All pregnant women are routinely screened for trichomonas Explanation: T. vaginalis infection in pregnant women is associated with adverse pregnancy outcomes, particularly premature rupture of membranes, preterm delivery, and delivery of a low birthweight infant. Although metronidazole treatment produces parasitologic cure, certain trials have shown no significant difference in perinatal morbidity following metronidazole treatment. Symptomatic pregnant women, regardless of pregnancy stage, should be tested and considered for treatment. Treatment of T. vaginalis infection can relieve symptoms of vaginal discharge in pregnant women and reduce sexual transmission to partners. If treatment is considered, the recommended regimen in pregnant women is metronidazole 500 mg orally twice a day for 7 days. Although perinatal transmission of trichomoniasis is uncommon, treatment also might prevent respiratory or genital infection of the newborn. Clinicians should counsel symptomatic pregnant women with trichomoniasis regarding the potential risks for and benefits of treatment and about the importance of partner treatment and condom use in the prevention of sexual transmission. The benefit of routine screening for T. vaginalis in asymptomatic pregnant women has not been established. However, screening at the first prenatal visit and prompt treatment, as appropriate, are recommended for pregnant women with HIV infection, because T. vaginalis infection is a risk factor for vertical transmission of HIV. Pregnant women with HIV who are treated for T. vaginalis infection should be retested 3 months after treatment.

An 18-year-old nulligravid woman presents to the student health clinic with a 4-week history of yellow vaginal discharge. She also reports vulvar itching and irritation. She is sexually active and monogamous with her boyfriend. They use condoms inconsistently. On physical examination, she is found to be nontoxic and afebrile. On genitourinary examination, vulvar and vaginal erythema is noted along with a yellow, frothy, malodorous discharge with a pH of 6.5. The cervix appears to have erythematous punctuations. There is no cervical, uterine, or adnexal tenderness. The addition of 10% KOH to the vaginal discharge does not produce an amine odor. Wet prep microscopic examination of the vaginal swabs is performed. What would you expect to see under microscopy? A. Branching hyphae B. Multinucleated giant cells C. Scant WBCs D. Flagellated, motile organisms E. Epithelial cells covered with bacteria

D. Flagellated, motile organisms Explanation: The symptoms and findings are most consistent with Trichomonas. Symptomatic patients report a frothy vaginal discharge that may be yellow, gray, or green. Vaginal pH is typically 6 to 7 and vulvar or vaginal mucosa is erythematous. The classical cervical findings include erythematous, punctate epithelial papillae, or "strawberry" appearance. Motile, flagellated protozoans are visible on wet prep microscopic examination. Branching hyphae is seen with yeast infections. Symptoms include vaginal burning, pruritus, and increased thick, white, curdy vaginal discharge. Multinucleated giant cells can be seen on Tzanck smear preparation of herpetic lesions. Genital herpes is typically described as multiple 1 to 3 mm painful vesicles. You would expect to see increased WBCs with Trichomonas. Epithelial cells covered with bacteria, also known as clue cells, are seen with bacterial vaginosis (BV). Typical findings of BV include pH greater than 4.5 as well as an increase thin, white, malodorous, fishy vaginal discharge coating the vaginal walls. The addition of 10% KOH to the vaginal discharge typically produces a fishy amine odor. BV does not usually cause vaginal inflammation.

A 30-year-old woman presents with a vaginal discharge and soreness. She had a cervical smear which was reported as showing severe dyskaryosis and Trichomonas vaginalis. A biopsy of the cervical lesion was reported as CIN III. Which is the SINGLE MOST appropriate management? A. Treat with metronidazole B. Repeat cervical cytology C. Cone biopsy D. Large loop excision of the transformation zone (LLETZ) E. Total abdominal hysterectomy

D. Large loop excision of the transformation zone (LLETZ) Explanation: The treatment of her incidental infection in not a priority, although she should be treated. The most appropriate treatment she should be offered is LLETZ.

A 28-year-old para 3 presents with vulval pruritus and burning. She reports dyspareunia and copious foul-smelling green vaginal discharge. On examination there is erythema of the vulva as well as petechiae of the upper vagina and cervix. What is the most likely diagnosis? A. Chlamydia B. Gonorrhoea C. Syphilis D. Trichomonas vaginalis E. Candidiasis

D. Trichomonas vaginalis Explanation: The history and examination are highly suggestive of trichomoniasis, although it would be important to assess for other STIs.

A 15-year-old girl attends the genitourinary medicine clinic complaining of vaginal itching and green vaginal discharge. She is sexually active with her 17-year-old boyfriend and uses the oral contraceptive pill. Speculum examination reveals haemorrhages on her cervix. A urine pregnancy test is negative. Considering the most likely diagnosis, what is the most appropriate first line antibiotic? A. Azithromycin 1 g once only B. Doxycycline 100 mg twice daily plus metronidazole 400 mg three times daily plus ofloxacin 400 mg twice daily for 7 days C. Doxycycline 100 mg twice daily for 14 days plus metronidazole 400 mg three times daily for 7 days D. Metronidazole 400 mg twice daily for 5 days E. Tinidazole 2 g once only

D. Metronidazole 400 mg twice daily for 5 days Explanation: Trichomonas vaginalis is a sexually transmitted infection of the lower genital tract caused by a protozoa. Symptoms include itching and inflammation of the vulva and vagina. There is often purulent vaginal discharge and the cervix may have the classic haemorrhages giving it the classic description of a 'strawberry cervix'. Swabs should be sent for culture and other sexually transmitted diseases should always be considered. Trichomonas is a rare cause of pelvic inflammatory disease, however contact tracing is necessary. First line treatment is metronidazole, either 2 g as a single dose or 400 mg twice daily for 5-7 days. Sexual intercourse should be avoided until treatment is completed in the patient and also the partner, if necessary. This patient should be advised that while the oral contraceptive pill is effective for preventing pregnancy, barrier methods should also be used to prevent sexually transmitted infections. Treatment with metronidazole would be safe in pregnancy.

A 22-year-old woman attends the sexual health clinic with offensive vaginal discharge. She has been in a stable relationship for the past 4 years. Triple swabs are taken. Wet-mount examination shows motile flagellated protozoa. What is the appropriate management option? A. Ciprofloxacin 500 mg oral single dose and referral to the genitourinary medicine clinic B. Cefotriaxone 500 mg IM injection stat followed by doxycycline 100 mg twice daily for 14 days C. Ofloxacin 400 mg twice daily and metronidazole 400 mg twice daily for 14 days D. Metronidazole 400 mg twice daily for 7 days E. Azithromycin 1 g stat

D. Metronidazole 400 mg twice daily for 7 days Explanation: Oral metronidazole 2 g as a stat dose or 400 mg twice daily for a week are both accepted regimens for treating trichomonal infection.

A previously healthy 21-yr-old woman has a profuse malodorous vaginal discharge. Examination shows greenish gray frothy discharge with fishy odor and patechial lesions on the cervix. There is no cervical motion tenderness. Her temperature is 37.5 C, blood pressure is 120/80, pulse is 60 and respirations are 16. Microscopic evaluation of the discharge is most likely to show which of the following. A. Clue cells B. Gram-negative diplococcic C. Gram-positive diplococcic D. Motile flagellated organisms E. Hyphae or pseudohyphae

D. Motile flagellated organisms Explanation: This patient has trichomoniasis. Trichomoniasis is caused by a motile, flagellated protozoan, Trichomonas vaginalis. The symptoms include a copious, malodorous ("fishy"), greenish-gray, "frothy" discharge. The vulvar and vaginal epithelium may be erythematous and edematous. Colposcopy may reveal petechial cervical lesions ("strawberry cervix"). A wet mount of the discharge often reveals motile trichomonads and polymorphonuclear leukocytes (PMNs). The treatment is metronidazole. Simultaneous treatment of the sexual partner reduces the risk of reinfection. "Clue cells" (choice A), vaginal squamous epithelial cells coated with coccobacillary organisms, are seen in bacterial vaginosis. The symptoms include a moderate amount of malodorous ("fishy"), white to gray, homogeneous vaginal discharge. An amine ("fishy") odor is present after mixing vaginal secretions with KOH. This is often called a positive whiff test. Saline preparations of the discharge reveal the "clue cells". The treatment is metronidazole. Simultaneous treatment of the sexual partner has not been shown to reduce recurrence. Gram-negative diplococci (choice B) are an indication of Neisseria gonorrhoeae. N. gonorrhoeae causes a mucopurulent cervical discharge in acute cervicitis and can lead to pelvic inflammatory disease (PID). PID is characterized by lower abdominal pain, fever, and cervical motion tenderness. Diagnosis is often made by Gram's stain of cervical secretions revealing gram-negative diplococci and polymorphonuclear leukocytes. Treatment is ceftriaxone IM once and doxycycline or azithromycin. The 2 latter drugs are given since concomitant chlamydial infection is common. Sexual partners must be treated. Gram-positive diplococci (choice C) are not a common cause of cervical discharge. Pseudohyphae or hyphae (choice E) is an indication of candidiasis. Vulvar pruritus, irritation, and a thick, white, cottage cheese-like discharge are the predominant symptoms. Diagnosis is made by KOH, saline, or Gram's stain evaluation of the vaginal fluid revealing fungi. Treatment is fluconazole PO or imidazole cream. Routine treatment of sexual partners is usually not indicated.

What is the most sensitive test for detection of Trichomonas vaginalis infection? A. Immunochromatographic capillary flow (dipstick) assay B. Laboratory culture C. Light-field microscopy D. Nucleic acid amplification test (NAAT) E. Trichomonas serum antibody test

D. Nucleic acid amplification test (NAAT) Explanation: NAATs offer the highest sensitivity for the detection of T. vaginalis. They should be the test of choice where resources allow and are becoming the current 'gold standard'. In-house polymerase chain reaction (PCR) tests have shown increased sensitivity in comparison with both microscopy and culture.

A 21-year-old woman presents with florid vaginitis with a profuse, yellow, irritating, frothy and offensive discharge. The MOST APPROPRIATE next step would be to A. Give the patient an imidazole cream, such as clotrimazole B. Culture for gonorrhea and Chlamydia C. Culture for candida and trichomonas D. Perform potassium hydroxide (KOH) smear and wet smear of vaginal discharge E. Obtain venereal disease reference laboratory (VDRL) or fluorescent treponema antibodies (FTA-ABS) for syphilis

D. Perform potassium hydroxide (KOH) smear and wet smear of vaginal discharge Explanation: The likely cause of the discharge observed are trichomonal vaginitis or bacterial vaginosis (gardenerella infection) To perform smears of the vaginal discharge (KOH and wet smear) is correct, will enable the correct diagnosis to be made and will allow effective therapy to be commenced. In general, therapy should not be commenced before the smears or swab is taken to make the correct diagnosis. Clotrimazole is an effective antithrush medication, but is not effective against trichomonas or gardnerella; the symptoms and signs are not those of gonococcal or chlamydial infection and syphilis infection is rarely associated with either trichomonal or gardnerella infection.

A 63-year-old woman is evaluated for a 3-week history of vaginal discharge. The discharge is described as yellowish and malodorous and is accompanied by burning and dyspareunia. She is postmenopausal and is sexually active with a new male partner. Her last Pap smear was 2 years ago and was normal. Medical history is significant for hypertension, and her only medication is hydrochlorothiazide. On physical examination, the patient is afebrile, blood pressure is 128/78 mm Hg, pulse rate is 72/min, and respiration rate is 12/min. BMI is 26. On pelvic examination, a frothy, yellowish discharge is present in the vaginal vault. The cervix is without lesions, although there is contact bleeding with speculum placement. There is no cervical motion tenderness or adnexal tenderness. The remainder of the examination is unremarkable. Laboratory studies show vaginal pH is 6.0; whiff test is negative. Saline microscopy is shown. Potassium hydroxide microscopy is negative. Testing for Chlamydia trachomatis and Neisseria gonorrhoeae is negative. Treatment with single-dose metronidazole is planned. Which of the following is the most appropriate additional intervention? A. Confirm infection with a nucleic acid amplification test B. Perform repeat Pap smear following treatment C. Test for cure following treatment D. Treat the patient's sexual partner E. Empiric treatment for chlamydia and gonorrhea

D. Treat the patient's sexual partner Explanation: The most appropriate additional intervention in this patient is to treat her partner. This patient has the characteristic features of frothy yellow discharge, burning, and dyspareunia associated with infection by Trichomonas vaginalis organisms, which are seen on microscopy. Trichomoniasis is the most common curable sexually transmitted infection (STI) worldwide and is evenly distributed among women of all age groups, unlike other STIs that predominate in younger people. It is caused by motile flagellated protozoa that infect the urogenital tract, causing inflammatory vaginitis and urethritis. Treatment with a single 2-g dose of metronidazole is associated with a high rate of cure and should be offered to all symptomatic women. It is important that sexual partners also be treated, even if asymptomatic, because of a high rate of reinfection; documentation of infection is not required before treatment in any partners. Once trichomoniasis is identified, testing for other STIs should be considered in both individuals. Trichomoniasis has been traditionally associated with a vaginal pH of greater than 4.5 and diagnosed by direct visualization of trichomonads on saline microscopy. However, the specificity of a high vaginal pH and the sensitivity of saline microscopy findings are low. Therefore, point-of-care vaginal swab rapid immunoassays and nucleic acid amplification tests (NAATs) for detection of T. vaginalis are increasingly considered to be the gold standard for diagnosis, and particularly when microscopy is not available. NAATs can be performed on a vaginal (or endocervical) swab, urine sample, or liquid-based Pap test specimens. In this patient in whom trichomonads are seen on microscopy, confirmatory testing with another assay is not necessary. Trichomoniasis causes an inflammatory vaginitis, and cervical contact bleeding may occur due to inflammation from the infection. This inflammation resolves with treatment of the infection, and a follow-up Pap smear to detect cervical pathology would not be required in the absence of additional symptoms or findings. Treatment of trichomoniasis with single-dose metronidazole is highly effective; therefore, testing for cure is not required in patients whose symptoms have resolved with therapy.

A 30-year-old woman presents to her gynecologist with complaints of vaginal itching and a frothy, yellow discharge. She also complains of painful urination. She admits to being sexually active with several men in the past two weeks. Cultures are negative for bacterial growth, but organisms are visible via a wet prep on low power. The most likely causal agent is A. Candida albicans B. Trichophyton rubrum C. Chlamydia trachomatis D. Trichomonas vaginalis E. Giardia lamblia

D. Trichomonas vaginalis Explanation: The clues are frothy, yellow discharge, itching, organisms identified on wet mount, bacterial cultures were negative. With Candida albicans (choice A), the discharge would have been white and creamy. Trichophyton (choice B) causes skin, hair, and nail infections and is a cutaneous fungus. Chlamydia trachomatis (choice C) would not be visible on wet mount and causes intracellular infection of epithelial cells. Giardia lamblia (choice E) is associated with diarrhea.

An 18-year-old woman presents to her physician with a few days' history of a thin, foul-smelling, vaginal discharge. She complains of vaginal itching and pain with urination and sex. Cervical examination reveals a bright red, "strawberry cervix." What is the most likely diagnosis? A. Genital herpes B. Vulvovaginal thrush C. Bacterial vaginosis D. Trichomoniasis E. Syphilis

D. Trichomoniasis Explanation: The most likely diagnosis is the sexually transmitted disease trichomoniasis, caused by the flagellate Trichomonas vaginalis. She should be treated with metronidazole. Her partner should also be treated to prevent reinfection. Trichomonas vaginalis causes trichomoniasis and is a motile, flagellated protozoan that does not form cysts. Infection is acquired by sexual transmission of Trichomonas trophozoites. In women, it causes a vaginitis characterized by itching, burning, dysuria, and a thin, malodorous discharge that can be clear, white, yellow, or green in color. In men, it may cause a mild urethritis. Diagnosis is made by visualization of motile trophozoites on wet mount. Treat with metronidazole. Interestingly, Trichomonas is the only protozoan that does not exist in cyst form because it never leaves the host environment (it is sexually transmitted and therefore has no need for a protective/dormant form).

A 20-year-old female college student has just returned from Spring break in Cancun and presents with a watery, foul-smelling vaginal discharge with severe itching and burning. After a pelvic examination, you also observe a strawberry cervix. You diagnose her with trichomoniasis and take a vaginal swab. What organism are you looking for? A. Eggs and excrement of Sarcoptes scabiei B. Cysts of Trichomonads C. Yeast cells with multiple buds D. Trophozoite with undulating membrane and polar flagella E. Spirochetes under dark-field examination

D. Trophozoite with undulating membrane and polar flagella Explanation: The most common diagnostic testing performed for trichomoniasisis is wet prep microscopy. Trichomonads are motile organisms with a single flagellum and can be seen moving in the preparation when viewed with a microscope. This test has been shown to be only 40%- 60% sensitive but is typically the most common testing method used due to convenience and low cost.

A 17-year-old never pregnant female was treated for trichomonas infection with a single 2-gram dose of metronidazole. She returns for a test of cure 2 months later. While a wet prep is negative her nucleic acid amplification test for trichomonas is positive. She denies any sexual activity since her initial treatment. What is the next step in management? A. No further treatment B. 2 grams of metronidazole by mouth once C. 2 grams of tinidazole by mouth once D. 2 grams of tinidazole by mouth daily for 7 days E. 500 mg of metronidazole by mouth twice a day for 7 days

E. 500 mg of metronidazole by mouth twice a day for 7 days Explanation: UK treatment protocol for non-response to standard TV therapy (having excluded re-infection and non-adherence): (A) If standard treatment failure has occurred with metronidazole 2 g single dose and reinfection is excluded, the patient (and their partner[s]) can be treated -----> • Repeat course of 7-day standard therapy. - Metronidazole 400-500 mg twice daily for 7 days (B) If this regimen fails -----> • Consider treatment with higher-dose course of nitroimidazole. - Metronidazole or tinidazole 2 g daily for 5-7 days or - Metronidazole 800 mg three times daily for 7 days) (C) If this regimen fails -----> • Testing of the organism for metronidazole and tinidazole susceptibility is recommended (if available). • Consider treatment with very high-dose course of tinidazole. - Tinidazole 1 g twice or three times daily, or 2 g twice daily for 14 days +/- intravaginal tinidazole 500 mg twice daily for 14 days. (D) If this regimen fails -----> • Consider other treatments with some reported success. - Paromomycin* intravaginally 250 mg once or twice daily for 14 days. - Furazolidone* intravaginally 100 mg twice daily for 12-14 days. - Acetarsol* pessaries 500 mg nocte for 2 weeks. - 6% Nonoxynol-9* pessaries nightly for 2 weeks. *The medicines suggested for use in treatment failure are unlicensed products and may not be readily available for purchase in the UK. The pharmacy purchasing department may be able to source some of these products from specialist manufacturers. At the time of writing this guideline, Acetarsol 500 mg pessaries and Nonoxynol-9 75 mg pessaries were available from Pharmarama International Ltd. The lead time for ordering these products for the first time may be up to 8 weeks.

A 24-year- old patient presents to your office with complaints of a 1-week history of vulvar itching and vaginal discharge. She endorses that she has had intercourse with two individuals in the past 6 months without the use of a condom. On physical examination, the vulva is edematous. Yellow-green discharge is present on the vaginal walls and at the cervical os. The cervix is strawberry red Wet smear of vaginal secretions shows epithelial cells, white blood cells (WBCs), and flagellate protozoa. You prescribe an antibiotic. What warning do you give about this particular treatment? A. Avoid sun overexposure B. Avoid taking on an empty stomach C. Avoid taking with grapefruit juice D. Avoid taking with oral contraceptive pills E. Avoid alcohol

E. Avoid alcohol Explanation: T. vaginalis is a flagellate protozoan that is transmitted by sexual contact and causes a yellow-green frothy discharge. In 2 % of the cases, T. vaginalis can cause petechiae or "strawberry patches" on the upper vagina and cervix. Diagnosis is made by wet smear of vaginal discharge. Treatment of trichomoniasis is oral metronidazole-major side effect is antabuse effect- alcohol may cause an upset stomach, vomiting, stomach cramps, headaches, sweating, and flushing (redness of the face).

A 19-year-old man returns home after his first semester at college. While away he had two female sexual partners and reports that he rarely used condoms. He is anxious about sexually transmitted infections and requests screening. A plan is made to perform testing for Chlamydia trachomatis and Neisseria gonorrhoeae using nucleic acid amplification testing performed on a urine specimen in addition to blood testing for syphilis and HIV. He mentions that he has read about trichomoniasis on the internet and asks whether he should be tested for this as well. Which one of the following would you recommend regarding testing this man for Trichomonas vaginalis? A. Urethral swab for Trichomonas vaginalis culture B. Urine sample for Trichomonas vaginalis culture C. Point-of-care (POC) testing for Trichomonas vaginalis D. Urine nucleic acid amplification testing (NAAT) for Trichomonas vaginalis E. Do not offer testing, as there is no clear benefit to screening asymptomatic men for Trichomonas vaginalis infection

E. Do not offer testing, as there is no clear benefit to screening asymptomatic men for Trichomonas vaginalis infection Explanation: There is no clear benefit to screening asymptomatic men for Trichomonas vaginalis infection and therefore routine screening for trichomonas infection in males in not recommended. In contrast, diagnostic testing for T. vaginalis infection may reasonably be performed in male patients presenting with a syndrome consistent with trichomoniasis. Neither the OSOM nor the Affirm VP III Point-of-care test is FDA-cleared for use with specimens obtained from men. In this situation, nucleic acid amplification testing (NAAT) is the preferred test as it is more sensitive than culture. In men, NAAT for T. vaginalis should be performed on urethral meatal swabs rather than on urine specimens, given superior sensitivity with the urethral swab (80% versus 39% in one study). Most men with T. vaginalis infection are asymptomatic. In a 2007 survey that evaluated male sex partners of women with trichomoniasis, 177 men were diagnosed with T. vaginalis infection and of these 76.8% were asymptomatic. Use of NAAT testing of rectal swabs for T. vaginalis in patients who practice receptive anal intercourse is not recommended.

An 18-year-old woman comes to the physician because of nausea, vomiting, and abdominal pain 1 hour after ingesting a glass of wine with dinner. Three days ago, she began antibiotic treatment for vaginitis after a wet mount preparation of vaginal discharge showed a motile protozoan. This patient most likely has been taking which of the following drugs? A. Ceftriaxone B. Chloroquine C. Clindamycin D. Ofloxacin E. Metronidazole

E. Metronidazole Explanation: The nitroimidazoles are the only class of antimicrobial medications known to be effective against T. vaginalis infections. Of these drugs, metronidazole and tinidazole have been cleared by FDA for the oral or parenteral treatment of trichomoniasis. Tinidazole is generally more expensive, reaches higher levels in serum and the genitourinary tract, has a longer half-life than metronidazole (12.5 hours versus 7.3 hours), and has fewer gastrointestinal side effects. Alcohol consumption should be avoided during treatment with nitroimidazoles. To reduce the possibility of a disulfiram-like (Antabuse effect) reaction, abstinence from alcohol use should continue for 24 hours after completion of metronidazole or 72 hours after completion of tinidazole.

A 30-year-old woman presents with a frothy green/creamy vaginal discharge. On examination, she is found to have an inflamed vulva and vagina. A high vagina swab demonstrates flagellates and clue cells. Which of the following is the first-line treatment option? A. Topical corticosteroids B. Benzyl benzoate C. Clotrimazole cream D. Mebendazole E. Metronidazole

E. Metronidazole Explanation: A greenish vaginal discharge which is associated with an inflamed vulva is most likely to be secondary to Trichomonas vaginalis. The presence of clue cells also suggests bacterial vaginosis. The treatment of choice for both conditions is metronidazole.

A 29-year-old woman presents to her obstetrician for her first antenatal clinic visit. She is feeling well and without complaint. Routine screening with nucleic acid amplification testing of urine demonstrates infection with Trichomonas vaginalis. Ultrasound demonstrates a healthy 7-week gestational age fetus. Testing for HIV is negative. After discussion of the potential benefits and risks of treatment, the decision is made to treat the trichomoniasis. In addition, her partner is referred for presumptive therapy. What regimen should be used for treatment of this pregnant woman with Trichomonas vaginalis infection? A. Metronidazole 2 grams orally in a single dose B. Tinidazole 2 grams orally in a single dose C. Nitazoxanide 500 mg orally twice a day for 3 days D. Tinidazole 500 mg orally twice a day for 3 days E. Metronidazole 500 mg orally twice a day for 7 days

E. Metronidazole 500 mg orally twice a day for 7 days Explanation: Infection with Trichomonas vaginalis in pregnant women presents risks, including greater risk of premature rupture of membranes, preterm delivery, and delivery of a preterm infant. Although treatment of T. vaginalis infection during pregnancy has not demonstrated improvement in perinatal morbidity, benefits of treatment are generally thought to outweigh the risks, and symptomatic women should be treated at diagnosis, although some clinicians have preferred to defer treatment until the second trimester. The British National Formulary advises against highdose regimens in pregnancy. Metronidazole crosses the placenta but has not been demonstrated to be teratogenic and is thought safe for use in all stages of pregnancy. Data from animal models show that tinidazole may pose some risk to the fetus and should be avoided during pregnancy. It is particularly important in pregnant women that all sexual partners receive prompt referral for evaluation and presumptive treatment of trichomoniasis.

A 39-year-old woman seeks care for malodorous yellow-green vaginal discharge. A wet mount of her vaginal secretions is examined and motile organisms are seen. Urine nucleic acid amplification testing for Chlamydia trachomatis and Neisseria gonorrhoeae infections are performed and both are negative. The diagnosis of trichomoniasis is made and the clinician decides to treat with a single 2-gram oral dose of metronidazole. The patient reports that she was treated for giardiasis with metronidazole 2 years prior and developed full body urticarial and facial edema. These symptoms resolved with diphenhydramine. At this point what is the next best step in the management of this patient's trichomoniasis? A. Nitazoxanide 500 mg twice daily for 3 days B. Paromomycin 500 mg 4 times daily for 10 days C. Treat with tinidazole 2 g single oral dose but counsel patient to abstain from all alcohol during tinidazole therapy D. Treat with metronidazole 2 g single oral dose but counsel patient to abstain from all alcohol during metronidazole therapy E. Pursue metronidazole desensitization therapy

E. Pursue metronidazole desensitization therapy Explanation: Treatment of Trichomonas vaginalis infection in a patient with intolerance or allergy to nitroimidazoles can be a challenge owing to the limited therapeutic options. Only the nitroimidazoles—metronidazole and tinidazole—are reliably effective for treatment of T. vaginalis infection. Agents such as nitazoxanide and paromomycin have demonstrated activity in vitro but do not have established clinical efficacy for T. vaginalis infections and should not be used as an alternative to standard therapy with either metronidazole or tinidazole. Fortunately, true allergy to metronidazole is uncommon. More commonly, patients develop common adverse symptoms during metronidazole therapy including nausea and headache. Concurrent consumption of alcohol can cause a disulfiram-like reaction, including nausea, flushing, sweating, dizziness, tachycardia, and headache; the disulfiram-like reaction should not be mistaken for a true allergic reaction. The patient in this clinical scenario is presenting with a syndrome most consistent with and IgE mediated hypersensitivity reaction. She still requires therapy with metronidazole or tinidazole and should undergo metronidazole desensitization therapy; several protocols have been used and this should be done in consultation with an expert. If metronidazole desensitization is not available, consultation with a specialist to determine other options, including use of furazolidone, paromomycin sulfate, and Betadine douche. Unfortunately, all of these options have high failure rates.

Which of the following is true about the trichomonas vaginalis? A. Has three to five flagella at the posterior end B. Has two nuclei C. Forms cysts D. Has a sucking disc E. Survives well in ordinary water, even when this is hot or contains chlorine or detergents

E. Survives well in ordinary water, even when this is hot or contains chlorine or detergents Explanation: T. vaginalis are pear-shaped with three to five flagella at the broad front end and another flagellum forming part of an undulating membrane at the posterior end. The organism is similar in size to a white blood cell (10-20 μm), and readily identified by flagella movement in wet preparations under a × 40 microscope objective. Trichomonas survives well in ordinary water, even when this is hot or contains chlorine or detergents. In the presence of a trichomonal infection the normal rod-shaped lactobacilli of the vagina adopt a round, coccoidal form.

A 32-year-old woman complains of an offensive clear yellow vaginal discharge, with associated vulval itch and soreness. She admits to beginning a relationship with a new partner some four weeks earlier. On examination her vulva looks slightly erythematous and there is a clear discharge that has a fishy odour.Which of the following is the most likely diagnosis? A. Bacterial vaginosis B. C. trachomatis C. Herpes simplex infection D. N. gonorrhoeae E. T. vaginalis

E. T. vaginalis Explanation: The clinical picture described fits entirely with infection with the anaerobic flagellated protozoan Trichomonas vaginalis. Trichomoniasis differs from non-infective vaginosis, with respect to a yellowish colour to the discharge and associated vulval irritation. A large dose of metronidazole (2 g as a single course), or a seven day course at lower dose is the treatment of choice. Patients should of course also be screened for other sexually transmitted infections. Partners should be identified and also screened for infection as men rarely exhibit symptoms of a T. vaginalis infection. The epithelial damage caused by T. vaginalis increases susceptibility to HIV virus infection and transmission. Whilst bacterial vaginosis is also associated with a discharge with a fishy odour, classically there is no soreness or irritation associated with it.

A 21-year-old sexually active college student presents to the clinic complaining of odorous green vaginal discharge and itchiness. Multiple oval flagellated motile organisms are seen on wet mount. She is prescribed an antibiotic treatment. While on therapy she goes to a party, where on consumption of one alcoholic beverage she experiences flushing, tachycardia, headaches, and vomiting. The same effects could be observed when alcohol is mixed with which drug? A. Ampicillin B. Erythromycin C. Glipizide D. Imipenem E. Tolbutamide

E. Tolbutamide Explanation: The patient consumed alcohol while taking metronidazole for her trichomoniasis. Metronidazole and tolbutamide both have disulfiram-like adverse effects when combined with alcohol use. These effects include flushing, tachycardia, headaches, and vomiting. Tolbutamide is a first-generation sulfonylurea that can also produce these effects. Answer A is incorrect. Ampicillin is a penicillin family antibiotic often used to treat gram positive infections or Listeria infections. It does not have disulfiram-like adverse effects. Answer B is incorrect. Erythromycin is a macrolide antibiotic and is not commonly used to treat trichomoniasis. It does not have disulfiram-like adverse effects. Answer C is incorrect. Glipizide is a second generation sulfonylurea that does not have the disulfiram-like adverse effects of first-generation sulfonylureas. Answer D is incorrect. Imipenem is a carbapenem- type antibiotic with a very broad spectrum of action. The main adverse effect is that it can cause seizures in rare cases. It does not have disulfiram-like adverse effects.

A 16-year-old has recently become sexually active. She complains of intense pruritus vulvae associated with an offensive discharge with a fishy odour. On examination she has a marked vulvo-vaginitis and a frothy greenish vaginal discharge. What is the most likely diagnosis? A. Candidal infection B. Contact dermatitis C. Psoriasis D. Enterobiasis E. Trichomoasis

E. Trichomoasis Explanation: The most likely diagnosis is the sexually transmitted disease trichomoniasis, caused by the flagellate protozoan trichomonas vaginalis which is typically asymptomatic in men, but in women it often produces an intensely irritating vaginal discharge with inflammation of the vulva, vagina and cervix. The discharge is often frothy and offensive. Metronidazole is the drug of choice for trichomonas vaginitis.

A 25-year-old woman at 20 weeks gestation is told that her infection can cause preterm birth but has very little other effect on the baby. What is the single most likely infective organism responsible for this clinical characteristics described? A. Cytomegalovirus B. Herpes simplex C. Hepatitis B D. Gonorrhoea E. Trichomonas

E. Trichomonas Explanation: Trichomonas vaginalis is associated with preterm birth and low birth weight. Little neonatal morbidity is associated with maternal T vaginalis.

A young woman sought help from her physician for a vaginal discharge that was frothy yellow. The mucosal surfaces were inflamed and tender. Laboratory tests discovered a protozoan etiology. Which of the following protozoa is known only in the trophozoite stage? A. Balantidium coli B. Entamoeba histolytica C. Giardia lamblia D. Toxoplasma gondii E. Trichomonas vaginalis

E. Trichomonas vaginalis Explanation: T. vaginalis is the most common form of trichomoniasis in humans. T. vaginalis is pear-shaped with a short undulating membrane and flagella. Cysts have not been found. Infected tissues are usually tender and inflamed and covered by a frothy yellow or cream-colored discharge in women. B. coli (choice A) is a large (60 × 45 μm) ciliated trophozoite. They are mostly involved in harmless intestinal infections, causing diarrhea. E. histolytica (choice B) is a common parasite in the large intestine of humans which has three stages of development: active amoeba, inactive cyst, and the intermediate precyst. G. lamblia (choice C) is a flagellate found in the duodenum and jejunum of humans that exists as trophozoites and infective cysts. T. gondii (choice D) is a coccidian protozoan the final host of which is the cat. Transmission to humans is via the oocyst stage.

A 42-year-old woman comes to the physician because of vaginal itch and discharge, dysuria, and dyspareunia. These symptoms have been steadily worsening over the past 3 days. Pelvic examination reveals an erythematous vagina and a thin, green, frothy vaginal discharge with a pH of 6. Microscopic examination of the discharge demonstrates the presence of a pear-shaped, motile organism. Which of the following is the most likely pathogen? A. Candida albicans B. Gardnerella vaginalis C. Herpes simplex virus D. Treponema pallidum E. Trichomonas vaginalis

E. Trichomonas vaginalis Explanation: This patient has the symptoms and signs most consistent with a Trichomonas vaginalis infection. Patients with T. vaginalis typically experience vaginal itch and discharge, dysuria, frequency and urgency of urination, and dyspareunia. However, a significant minority (around 20%) of patients infected with T. vaginalis will be asymptomatic. The key finding to diagnose the infection is the presence of motile, pear shaped, flagellated organisms on the normal saline, wet-mount smear preparation. These organisms will be smaller than the surrounding vaginal epithelial cells but larger than white blood cells. The treatment for T. vaginalis is metronidazole. Candida albicans (choice A) is a common cause of vaginitis. We know from the findings, however, that this patient does not have a Candida infection. Her discharge is not consistent with Candida infection. Candida typically causes a thick, white ("cottage-cheese") discharge with a pH of 4 to 5. Also, microscopic examination demonstrates the organism T. vaginalis and not the pseudohyphae seen with a Candida infection. Gardnerella vaginalis (choice B) is a common organism in bacterial vaginosis, in association with increased levels of anaerobic bacteria. The discharge in bacterial vaginosis can appear similar to that caused by T. vaginalis. However, bacterial vaginosis is usually characterized by a strong odor, and irritation of the vaginal epithelium is usually not seen. Furthermore, this patient has an identifiable organism on wet-mount. Herpes simplex virus (choice C) infection is characterized by vesicles and ulcers and an extremely tender vulva and vaginal area. This patient has no vesicles or ulcers and has an obvious organism on wet-mount. Treponema pallidum (choice D) is the organism that causes syphilis. Primary infection with T. pallidum is characterized by a painless chancre on the vulva, vagina, or cervix. The organism is identified on dark-field microscopy and not wet-mount preparation.

A 32-year-old nulligravid woman comes to the physician because of a 6-week history of persistent foul-smelling vaginal discharge and vaginal itching. Her symptoms have not improved despite 2 weeks of treatment with over-the-counter antifungal medications and fluconazole. She has been sexually active and monogamous with her boyfriend during the past year, and they use condoms consistently. Examination shows excoriated labia and erythematous vaginal mucosa. There is a frothy, watery-gray discharge in the posterior vaginal vault. A wet mount preparation of the discharge shows numerous multi-flagellated organisms the size of erythrocytes. Which of the following is the most likely causal organism? A. Haemophilus ducreyi B. Neisseria gonorrhoeae C. Pseudomonas aeruginosa D. Treponema pallidum E. Trichomonas vaginalis

E. Trichomonas vaginalis Explanation: Trichomoniasis (trich) is an infectious disease caused by the parasite Trichomonas vaginalis. Between 50% of women and men do not have symptoms when infected. When symptoms do occur they typically begin 5 to 28 days after exposure. Symptoms can include itching in the genital area, a bad smelling thin vaginal discharge, burning with urination, and pain with sex. Having trichomoniasis increases the risk of getting HIV/AIDS. It may also cause complications during pregnancy. Trichomoniasis is a sexually transmitted infection (STI) which is most often spread through vaginal sexual intercourse. People who are infected may spread the disease even when symptoms are not present. Diagnosis is by finding the parasite in the vaginal fluid using a microscope, culturing the vagina or urine, or testing for the parasite's DNA. If present other sexually transmitted infections should be tested for. Methods of prevention include not having sex, using condoms, and being tested for STIs before having sex with a new partner. Trichomoniasis can be cured with antibiotics, either metronidazole or tinidazole. Sexual partners should also be treated. About 20% of people get infected again within three months of treatment.

A 25-year-old woman attends the GUM clinic complaining of increased vaginal discharge which has an unpleasant odour. She says sexual intercourse with her partner is uncomfortable. A swab is taken and sent to the lab. On direct microscopy a flagellated protozoan is seen. Which is the most likely pathogen? A. Candida albicans B. Chlamydia trachomatis C. Gardnerella vaginalis D. Neisseria gonorrhoeae E. Trichomonas vaginalis

E. Trichomonas vaginalis Explanation: Trichomoniasis is a sexually transmitted infection caused by the flagellated protozoan Trichomonas vaginalis, which invades superficial epithelial cells of the vagina, urethra, glans penis, prostate and seminal vesicles. Affected females present with an offensive frothy greeny-grey discharge, vulval soreness, dyspareunia, dysuria, vaginitis and vulvitis, although some are asymptomatic. On examination, the cervix may have a punctate erythematous (strawberry) appearance. Males are mostly asymptomatic. Diagnosis is by direct microscopy or culture of vaginal exudate. Treatment is with metronidazole. Gonorrhoea is caused by the Gram-negative diplococcus Neisseria gonorrhoeae infecting the mucosal surfaces of the genitourinary tract, rectum and pharynx. The majority of females are asymptomatic (70%). Around 85-90% of cases involve the cervix but only 10% of these cases have a significant increase in vaginal discharge. Seventy percent of cases involve the urethra and again it is generally asymptomatic although dysuria and urinary frequency may be seen. The vagina is not infected. Complications include Bartholin's abscess and gonococcal salpingitis with irreversible tube damage. Infected males present with dysuria, frequency and/or a mucopurulent discharge after 3 to 5 days, coupled with urethritis and meatal oedema. Disseminated gonococcal infection occurs in ,1% cases and causes pyrexia, a vasculitic rash and polyarthritis. Culture sensitive antibiotics are used for treatment. Chlamydia is caused by the oculogenital serovars D-K of Chlamydia trachomatis. Infection tends to be asymptomatic, although there can be increased vaginal discharge (30%), dysuria and urinary frequency. On examination, a 'cobblestone' appearance of the cervix may be noted. Ascending infection can cause salpingitis and, if it enters the abdominal cavity, perihepatitis (Fitz-Hugh-Curtis syndrome) which leads to right upper quadrant pain and tenderness. Chlamydia is a major cause for infertility and increases the possibility of ectopic pregnancy. In males, symptoms include mucopurulent discharge and dysuria (asymptomatic in 25%). Epididymo-orchitis is a complication. Diagnosis of chlamydia infection is by urine antigen detection or vaginal swab culture. Treatment is with doxycycline. Bacterial vaginosis (BV) is not a sexually transmitted infection. It is an infection caused by mixed anaerobic flora (commonly Gardnerella vaginalis and Mycoplasma hominis). It is often asymptomatic, but can cause a creamy-grey discharge with a fishy odour. There is no itching. The risk of preterm delivery and late miscarriage is doubled if BV is present in early pregnancy, however treatment does not reduce the perinatal risks so should only be used in symptomatic women. Diagnosis is by microscopy and treatment is with a single dose of metrinidazole (2 g). Candidiasis (thrush) is caused by yeasts, particularly Candida albicans and Candida glabrata, and produces vulval pruritis, burning, swelling and dyspareunia. White discharge and plaques are seen in the vagina with redness of the vulva and labia minora. Candidiasis is seen more commonly with pregnancy, tissue maceration, diabetes mellitus, HIV infection, and use of antimicrobial agents and immunosuppressive drugs. Diagnosis is confirmed by culture, and treatment is with antifungals, e.g. topical imidazoles (Canesten) or oral fluconazole. Chlamydia, from Greek chlamys = cloak (as chlamydia is often 'cloaked', i.e. asymptomatic). Candida, from Latin candidus = clear and white. Thomas Fitz-Hugh Jr, American physician (1894-1963). Arthur Curtis, American gynaecologist (1881-1955).

Which of the following protozoans is transmitted primarily by the motile trophozoite form? A. Balantidium coli B. Entamoeba histolytica C. Giardia lamblia D. Taenia solium E. Trichomonas vaginalis

E. Trichomonas vaginalis Explanation: Protozoans transmitted by the fecal-oral route are transmitted in the cyst form, which survives stomach acid. Only the sexually transmitted Trichomonas vaginalis is transmitted in the motile form. Taenia solium is not a protozoan, but a flatworm.

A young woman reports to her physician with possible urinary tract infection. The doctor finds the vagina and cervix tender, inflamed, and covered with a frothy yellow discharge. Which of the following protozoa described in this case is known to exist only in the trophozoite stage? A. Balantidium coli B. Entamoeba histolytica C. Giardia lamblia D. Toxoplasma gondii E. Trichomonas vaginalis

E. Trichomonas vaginalis Explanation: Trichomonas vaginalis is the only protozoan listed where the trophozoite is the diagnostic and infective stage that feeds on the mucosal surface of the vagina (bacteria and leukocytes). The trophozoite possesses a short, undulating membrane (one-half of the body) and four anterior flagella. No cyst stage exists. A persistent vaginitis with a frothy and foul-smelling discharge with burning, itching, and increased frequency of urination is common. Balantidium coli is a large ciliated protozoan parasite. Entamoeba histolytica is an intestinal amoeba. Giardia lamblia is a flagellated protozoan found in the duodenum and jejunum of humans. Toxoplasma gondii is a coccidian protozoan especially important in congenital infection when the pregnant woman becomes infected during pregnancy.

A woman is referred to the genitourinary medicine clinic with a three-day history of vaginal itching, offensive vaginal discharge and dysuria. A speculum examination is performed and the cervix is noted to be inflamed. There is a moderate amount of frothy offensive discharge and swabs are sent to the laboratory. The wet film of the high vaginal swab yields a positive result. What is the most likely organism to be detected? A. Candida albicans B. Chlamydia trachomatis C. Neisseria gonorrhoea D. Treponema pallidum E. Trichomonas vaginalis

E. Trichomonas vaginalis Explanation: Trichomonasis is a genital infection caused by the protozoon-Trichomonas vaginalis. Transmission is by sexual contact where Flagellated protozoa infects the vaginal epithelium and proliferates when vaginal pH is >5. Prevalence is 2-3% depending on population but higher amongst multiple sexual partners, unprotected sexual intercourse and presence of other STIs. Clinically it is presented by green/yellow (sometimes 'frothy') offensive discharge, vaginal soreness, pruritis vulvae, dysuria, dyspareunia (note: may be asymptomatic), PV discharge, vulval/vaginal erythema, and 'strawberry cervix' (diffuse/patchy macular erythematous cervical lesion) may be noted. Trichomonasis is strongly associated with the presence of other STIs, PID, and transmission HIV. Diagnosis is made by wet mount (shows presence of flagellated protozoa), HVS, and requires full STI screen. Metronidazole is effective in treating trichomoniasis with a cure rate of 95%. Contact tracing and treatment of sexual partner is recommended.

A 21-year-old woman comes to the genitourinary medicine clinic complaining of vaginal discharge. Vaginal high-swab results show that she has bacterial vaginosis. She asks you about treatment of her sexual partner. Which infection needs treatment of an asymptomatic sexual partner? A. Bacterial vaginosis B. Candidiasis C. Human papilloma virus D. Herpes simplex virus E. Trichomonas vaginalis

E. Trichomonas vaginalis Explanation: • Current sexual partners of women diagnosed with Trichomonas vaginalis (TV) should be offered a full sexual health screen and should be treated for TV irrespective of the results of their tests. • Concurrent treatment of all sexual partners is essential for symptomatic relief, microbiological cure, and to prevent reinfections or transfer to a new partner. • Sexual partners are advised to abstain from intercourse until they and their sexual partners have been cured. • Oral nitroimidazole drugs (e.g. metronidazole) are effective in treating trichomoniasis. • In the management of BV, testing and treatment of male sexual partners is not indicated. • In herpes simplex virus (HSV), symptomatic sexual partners should receive the same treatment. Asymptomatic sexual partners should be offered type-specific serological testing for HSV infection.

A young woman presents to the genitourinary medicine clinic. After a vaginal swab was taken from the posterior fornix, microscopic examination reveals a trichomonas vaginalis. Trichomoniasis is transmitted by which stage? A. Cyst B. Oocyst C. Egg D. Sporozoite E. Trophozoite

E. Trophozoite Explanation: Trichomonas vaginalis exists only as a trophozoite; no cyst stage has been identified. A cyst (A), oocyst (B) and sporozoite (D) are stages involved in transmitting other protozoan infections, and an egg (C) is the means of transmission in a number of helminth infections.

A young woman goes to her doctor with a variety of medical complaints. Her mucosal surfaces are tender, inflamed, eroded, and covered with a yellow discharge. For one problem, the laboratory reports the isolation of a pear-shaped protozoan with a whip-like motility. This organism is observed in which of the following? A. Biopsied muscle B. Blood C. Duodenal contents D. Sputum E. Vaginal secretions

E. Vaginal secretions Explanation: E. Vaginal secretions 50 Trichomonas vaginalis, an odd-looking protozoan, moves as a whip-like, almost darting motion. Trichomonasis, a bothersome vaginal infection, can be diagnosed by observing this organism in a wet mount of vaginal secretions. It may be washed out in the urine as well. Trichomonas vaginalis can be grown in special media, and there are now several products available for direct detection of the organism.

A 20-year-old woman presents to her gynecologist complaining of several days of vaginal itching and increased vaginal secretions that have an unpleasant odor. She denies any recent fever, back pain, hematuria, or vaginal bleeding. She has been sexually active with multiple sexual partners and rarely uses protection. On examination she has a moderate amount of frothy green discharge. Amine "whiff" test of the discharge is negative, and the pH of the discharge is 6. Multiflagellated organisms are seen on microscopy. Which of the following is the most likely diagnosis? A. Bacterial vaginosis B. Neisseria gonorrhoeae infection C. Syphilis D. Trichomoniasis E. Vaginal candidiasis

Trichomonas vaginalis is a protozoan with multiple flagella that causes a sexually transmitted vaginitis. Most commonly occurring symptoms include increased vaginal discharge with an unpleasant odor, dysuria, and vaginal pruritus. Infection with Trichomonas causes an increase in vaginal pH to >4.5 (normal vaginal pH is 3.8-4.2). Treatment with a single dose of 2 g of metronidazole is effective. The symptoms, increased vaginal pH, and flagellated organisms seen on wet mount establish the diagnosis. Answer A is incorrect. Bacterial vaginosis is caused by an imbalance in vaginal flora, with reduced numbers of lactobacilli and increased proportions of bacteria such as Gardnerella, Mobiluncus, or Peptostreptococcus species. Signs of infection include a thin, white vaginal discharge, vaginal pH >4.5, fishy odor on 10% potassium hydroxide "whiff" test, and clue cells on saline mount microscopy. Flagellated organisms would not be seen in bacterial vaginosis. Answer B is incorrect. Patients suffering from Neisseria gonorrhoeae cervicitis often complain of vaginal pruritus and discharge. However, up to 50% of patients may not manifest any symptoms at all. Diagnosis is established by identification of N. gonorrhoeae on chocolate agar culture or by DNA probe testing. The potassium hydroxide "whiff" test would be negative and no flagellated organisms would be seen. Answer C is incorrect. The first manifestation of syphilis, caused by infection with Treponema pallidum, is a painless primary chancre sore that appears 10-60 days after the infecting sexual contact. Several weeks later, symptoms of secondary syphilis can occur, which include constitutional symptoms, a maculopapular rash on the hands and feet, and fl at, broad condyloma lata. Vaginal symptoms do not result from syphilis, and flagellated organisms would not be seen. Treponema obtained from syphilitic lesions can be visualized with dark field microscopy. Answer E is incorrect. Patients with candidiasis often complain of a thick vaginal discharge, vaginal pruritus, or dysuria. Signs of candidiasis include vaginal pH in the range of 4-5, and budding hyphae or spores when examined on a slide treated with 10% potassium hydroxide preparation, which lyses the cells in the sample and makes the yeast easier to visualize. Clue cells are not seen in candidiasis.


संबंधित स्टडी सेट्स

Chapter 34: Assessment and Management of Patients with Inflammatory Rheumatic Disorders

View Set

Marketing: An Introduction - Chapter 5

View Set

Combo with "APUSH Chapter 9" and 6 others

View Set